Sie sind auf Seite 1von 202

N ationa: Board o f M ed ica l Examiners - G o o g le C hrom e

[ o

| 0

h ttp s://w w w .sta rtte st.e o m /a p i/5 .l.1.0/TTDStart.aspx?SVC=f3fd94ad-4a91-4962-b764-2d4caccal061

Exam Section 1: Item 1 o f 50 M ark

N ational Board o f M edical E xam iners C om prehensive Basic Science Self-Assessment

T im e R em aining: 4 hr 19 m in 0 sec

1. A 64-year-old woman comes to the physician because of a 6-month history of heartburn and difficulty swallowing solid food and liquid. She has not had nausea or vomiting. She also has a 2-year history of pain and swelling of her wrists and hands. She has not seen a physician in 5 years. She takes no medications. She does not smoke. Physical examination shows no wrinkles on the face and neck. Her vital signs are within normal limits. There is synovial thickening of the wrists bilaterally. A photograph of the left hand is shown. Abdominal examination shows no abnormalities. Which of the following sets of findings is most likely on esophageal manometry?

Esophageal Peristalsis
A) B) C) D) E) F) Increased Increased Normal Normal Decreased Decreased

Lower Esophageal Sphincter Tone


increased decreased increased decreased increased decreased

r -'

National Board o f Medical Examiners - Google Chrome https://www.starttest.com/api/5.1.1.0/ITDStart.aspx?SVC=f3fd94ad-4a91-4962-b764-2d4caccal061

Exam Section 1: Item 2 o f 50 I Mark

N ational Board o f M edical E xam iners C om prehensive Basic Science Self-Assessment

T im e R em aining: 4 hr 16 m in 41 sec

2. A 31-year-old woman is evaluated because of a 1-year history of infertility. Her hysterosalpingogram is shown. The most likely cause of these findings is malformation of which of the following embryonic tissues? A) Germ cells B) Gonadal ridges C) Mesonephric (wolffian) ducts D) Paramesonephric (mullerian) ducts E) Urogenital folds F) Urogenital sinus "S T s T -

Previous

Lab Values

Review

Pause 10:49 AM 28/08/2012

r -'

National Board o f Medical Examiners - Google Chrome https://www.starttest.com/api/5.1.1.0/TTDStart.aspx?SVC=f3fd94ad-4a91-4962-b764-2d4caccal061

Exam Section 1: Item 3 o f 50 I Mark

N ational Board o f M edical E xam iners C om prehensive Basic Science Self-Assessment

T im e R em aining: 4 hr 16 m in 4 sec

3. A 52-year-old man with a family history of renal failure comes to the physician because of abdominal pain over the past 6 months. He has been treated in the past for renal calculi and hypertension. Physical examination shows bilateral palpable masses in the upper abdomen. Urinalysis shows crenated erythrocytes and mild proteinuria. A CT scan of the abdomen identifies the cause of the patient's problem. Which of the following is the most likely diagnosis? A) Adrenal adenoma B) Glomerulonephritis O C) Henoch-Schonlein purpura O D) Polycystic kidney disease E) Renal cell adenoma

N ationa: Board o f M ed ica l Examiners - G o o g le C hrom e

[ o

| 0

h ttp s://w w w .sta rtte st.e o m /a p i/5 .l.1.0/TTDStart.aspx?SVC=f3fd94ad-4a91-4962-b764-2d4caccal061

Exam Section 1: Item 4 o f 50 M ark

N ational Board o f M edical E xam iners C om prehensive Basic Science Self-Assessment

T im e R em aining: 4 hr 13 m in 31 sec

4. A 55-year-old man comes to the physician because of two episodes of painless, blood-tinged urine during the past week. His last episode was 2 days ago. He has smoked 1 pack of cigarettes daily for 35 years. Vital signs are within normal limits. Physical examination shows no abnormalities. Urinalysis shows normal findings except for microscopic blood. Which of the following is the most likely diagnosis? O A) Bladder cancer B) Bladder diverticulum C) Interstitial cystitis D) Nephrolithiasis O E) Urinary tract infection

Previous

Next

Lab Values

Review

Help

Pause

Cl

Hu

il

4
1,5

10:52 AM 28/08/2012

N ationa: Board o f M ed ica l Examiners - G o o g le C hrom e

[ o

| 0

h ttps://w vvw .starttest.eom /api/5.l.1.0/ITDStart.aspx?SVC=f3fd94ad-4a91-4962-b764-2d4caccal061

Exam Section 1: Item 5 o f 50 M ark

N ational Board o f M edical E xam iners C om prehensive Basic Science Self-Assessment

T im e R em aining: 4 hr 12 m in 14 sec

5. A mouse melanoma, B-16, is highly metastatic. A clone of the B-16 melanoma cells is injected intravenously into the tail vein of mice, and 3 weeks later tumor nodules are found in the lungs, liver, and ovaries. When an antibody to a liver cell surface antigen is injected 1 hour prior to injection of the B-16 melanoma cells, formation of tumor nodules in the liver is prevented, but tumor nodules develop in the ovaries and lungs. In vitro studies do not show any cytotoxic effect of the antibody on B-16 or hepatic cells. Which of the following processes is most likely to be affected by the antibody? O A) Homing B) Initiation O C) Invasion O D) Motility O E) Progression O F) Promotion

o o 31l ? $ Tpr F
Previous Next

Lab Values

Review

Help

Pause

C l
il

Hu

^4
^

1,5

10:53 AM 28/08/2012

r -'

National Board o f Medical Examiners - Google Chrome https://www.starttest.com/api/5.1.1.0/TTDStart.aspx?SVC=f3fd94ad-4a91-4962-b764-2d4caccal061

Exam Section 1: Item 6 o f 50 I Mark

N ational Board o f M edical E xam iners C om prehensive Basic Science Self-Assessment

T im e R em aining: 4 hr 6 m in 13 sec

6. A retrospective study is conducted to assess the incidence and outcome of subarachnoid hemorrhage in a midwestern state. The annual incidence of this condition is 15 per 100,000 people, and the case fatality rate is 40%. Based on these data, which of the following represents the annual mortality rate for subarachnoid hemorrhage per 100,000 people in this state? A) 0.4 x 15 B) 0.4 x 100,000 C) 0.6 x 15 D) 0.6 x 100,000

r -'

National Board o f Medical Examiners - Google Chrome https://www.starttest.com/api/5.1.1.0/ITDStart.aspx?SVC=f3fd94ad-4a91-4962-b764-2d4caccal061

Exam Section 1: Item 7 o f 50 I Mark

N ational Board o f M edical E xam iners C om prehensive Basic Science Self-Assessment

T im e R em aining: 3 hr 51 m in 22 sec

7. A 24-year-old woman comes to the physician because of a 1-day history of pain in her left leg while walking. Her temperature is 37C (98.6F), pulse is 84/min, and blood pressure is 135/78 mm Hg. Examination of the left lower extremity shows only a markedly decreased pulse in the left lower extremity. An intermittent faint diastolic heart murmur is heard at the apex. Angiography of the left lower extremity shows complete obstruction of the distal left femoral artery by what appears to be a thrombus. The occlusion is removed using a catheter. Microscopic examination of the removed specimen shows scattered mesenchymal cells in an abundant extracellular matrix. Echocardiography confirms a diagnosis of cardiac tumor. This patient most likely has which of the following types of cardiac tumors? O A) Fibroelastoma O B) Fibroma C) Hamartoma O D) Hemangioma O E) Myxoma

N a tio n a i Board o f M ed ica l Examiners - G o o g le C hrom e

[ o

| 0

h ttp s://w w w .sta rtte st.e o m /a p i/5 .l.1.0/TTDStart.aspx?SVC=f3fd94ad-4a91-4962-b764-2d4caccal061

Exam Section 1: Item 8 o f 50 M ark

N ational Board o f M edical E xam iners C om prehensive Basic Science Self-Assessment

T im e R em aining: 3 hr 49 m in 1 sec

8. An 18-year-old woman with mild mental retardation is brought to the physician because of a 3-day history of decreased ability to see in reduced light. She has a lifelong history of chronic diarrhea. Two years ago, she developed a lack of muscle control of her arms and legs, and generalized weakness. Her 16-year-old brother has had similar symptoms. Ophthalmologic examination shows bilateral retinitis pigmentosa. There is ataxia and loss of deep tendon reflexes. Laboratory studies show erythrocytes with spiny projections and a serum total cholesterol concentration of 40 mg/dL. Which of the following apolipoproteins is most likely deficient in this patient? A) Apo A-l B) Apo A-ll C) Apo B D) Apo C E) Apo E

o Previous

o Next $

@ Lab Values

Review

Help

Pause

T p r

F 3 1 l

. V

11:16 AM 28/08/2012

r -'

National Board o f Medical Examiners - Google Chrome https://www.starttest.com/api/5.1.1.0/TTDStart.aspx?SVC=f3fd94ad-4a91-4962-b764-2d4caccal061

Exam Section 1: Item 9 o f 50 I Mark

N ational Board o f M edical E xam iners C om prehensive Basic Science Self-Assessment

T im e R em aining: 3 hr 47 m in 43 sec

9. A 21 -year-old African American man with sickle cell disease is brought to the emergency department 2 hours after the sudden onset of severe pain in his abdomen and knees. He has had six similar episodes within the past 4 years; he was admitted to the hospital each time for treatment of the pain. His course has been complicated by two episodes of pneumonia and a small cerebral infarction in the past 2 years. The patient asks the physician for extra pain medication to "kill me because I can't take it anymore." Which of the following initial actions by the physician regarding the patient's request is most appropriate? A) Contact the hospital ethics committee B) Restrain the patient C) Reevaluate the patient after his pain has been controlled D) Administer general anesthesia for sedation O E) Comply with the patient's request

r -'

National Board o f Medical Examiners - Google Chrome https://www.starttest.com/api/5.1.1.0/ITDStart.aspx?SVC=f3fd94ad-4a91-4962-b764-2d4caccal061

Exam Section 1: Item 10 o f 50 I Mark

N ational Board o f M edical E xam iners C om prehensive Basic Science Self-Assessment

T im e R em aining: 3 hr 40 m in 18 sec

10. A 40-year-old woman comes to the physician for genetic counseling because of a family history of muscle weakness. She says that she has not sought treatment because her symptoms are not severe, but her children have profound muscle weakness and fatigue. Sequence analysis of mitochondrial DNA shows a mutation in 50% of the mother's mitochondrial DNA and in 100% of each of her children's mitochondrial DNA. A pedigree is shown. Which of the following genetic principles best explains the findings in this family? A) Anticipation

O B) Genetic drift O C) Heteroplasmy D) Imprinting E) Pseudodominant inheritance

f i l l
1 2 3
100% 100% 100%

100%

O
#

U naffected fem ale Unaffected male A ffected fem ale A ffected m ale

o Previous

o Next $

@ Lab Values

Review

Help

Pause -a ^ il

T p r

F 3 1 l

4
1,5

11:25 AM 28/08/2012

N ational Board o f Medical Examiners - Google Chrome Q h ttps://w w w .sta rttest.eom /a p i/5 .l.1.0/TTDStart.aspx?SVC=f3fd94ad-4a91-4962-b764-2d4caccal061

'0

! & !

Exam Section 1: Item 11 o f 50 H Mark

National Board o f Medical Examiners Comprehensive Basic Science Self-Assessment

Tim e Remaining: 3 hr 37 min 44 sec

11. During an experiment, a cuff occluder is placed on the left renal artery of a normal rat and inflated to cause a 90% decrease in the cross-sectional area of the artery. A catheter is placed in the left and right renal veins to sample the effluent. The concentration of which of the following substances will most likely be higher in the effluent from the left renal vein than that from the right renal vein? O A )A D H (vasopressin) O B) Aldosterone 0 C) Endothelin 0 D) Epinephrine o E) Renin

Previous

Lab Values

Review

Help

Pause

O
* ..ill ><>

fTi

11:28 AM 28/08/2012

r-

N a tio n a l Board o f M ed ica l Examiners - G o o g le C hrom e

[ a

j (P

Q h ttp s://w w w .sta rtte st.e o m /a p i/5 .l.1.0/TTDStart.aspx?SVC=f3fd94ad-4a91-4962-b764-2d4caccal061

Exam Section 1: Item 12 o f 50 Mark

National Board o f Medical Examiners Comprehensive Basic Science Self-Assessment

Tim e Remaining: 3 hr 37 min 35 sec

12. A previously healthy 32-year-old man is brought to the emergency department because of a 1-day history of intermittent flashing spots and blurred vision, vomiting, confusion, and difficulty walking. He is a painter and says that yesterday he cleaned up a spilled bottle of paint thinner. Ophthalmologic examination shows dilated pupils with hyperemia of the optic disc and retinal edema. Physical examination shows tachypnea. Serum studies show an anion gap metabolic acidosis. The most appropriate pharmacotherapy for this patient is most likely to inhibit the activity of which of the following? A) Alcohol dehydrogenase B) Aldehyde dehydrogenase C) Formaldehyde dehydrogenase D) Lactate dehydrogenase O E) Pyruvate dehydrogenase

r -'

National Board o f Medical Examiners - Google Chrome https://www.starttest.com/api/5.1.1.0/TTDStart.aspx?SVC=f3fd94ad-4a91-4962-b764-2d4caccal061

Exam Section 1: Item 13 o f 50 I Mark

N ational Board o f M edical E xam iners C om prehensive Basic Science Self-Assessment

T im e R em aining: 3 hr 35 m in 50 sec

13. A 48-year-old man is brought to the emergency department because of difficulty opening his mouth and stiffness of his neck, shoulders, and back for 4 hours. He sustained a puncture wound to his arm 1 week ago. He has received no medical care for 30 years. Physical examination shows trismus and opisthotonos. Which of the following best describes the process in this patient? A) Blockade of inhibitory neurotransmitter release B) Cell death in central nervous system neurons C) Cell death in muscles D) Cell death in peripheral nervous system neurons O E) Demyelinization

r -'

National Board o f Medical Examiners - Google Chrome https://www.starttest.com/api/5.1.1.0/TTDStart.aspx?SVC=f3fd94ad-4a91-4962-b764-2d4caccal061

Exam Section 1: Item 14 o f 50 I Mark

N ational Board o f M edical E xam iners C om prehensive Basic Science Self-Assessment

T im e R em aining: 3 hr 35 m in 34 sec

14. A 76-year-old woman is brought to the emergency department because of a 3-hour history of severe headache, slurred speech, and confusion. She has a 2-year history of atrial fibrillation. She has difficulty understanding and answering questions, but she is cooperative. Ophthalmologic examination shows hemianopia and a tendency to gaze to the right. The pupils are normal sized and reactive to light. Neurologic examination shows left-sided numbness and paralysis that are more severe in the face and upper extremity than in the lower extremity. Which of the following arteries is most likely involved in this patient's condition? O A) Anterior cerebral O B) Anterior spinal O C) Middle cerebral O D) Posterior cerebral O E) Vertebral

r -'

National Board o f Medical Examiners - Google Chrome https://www.starttest.com/api/5.1.1.0/ITDStart.aspx?SVC=f3fd94ad-4a91-4962-b764-2d4caccal061

Exam Section 1: Item 15 o f 50 I Mark

N ational Board o f M edical E xam iners C om prehensive Basic Science Self-Assessment

T im e R em aining: 3 hr 34 m in 19 sec

15. A 54-year-old woman comes to the physician because of a 5-day history of severe mid-back pain. Physical examination shows point tenderness over the T6 vertebra. Serum studies show a calcium concentration of 13.4 mg/dl_; urinalysis shows Bence Jones proteins. Which of the following is the most likely cause of this patient's hypercalcemia? A) Excessive parathyroid hormone production B) Excessive parathyroid hormone-related protein production C) Increased fractional calcium gastrointestinal absorption O D) Local interleukin-1 (IL-1) and tumor necrosis factor effects E) Unregulated 1,25-dihydroxycholecalciferol production

r -'

National Board o f Medical Examiners - Google Chrome https://www.starttest.com/api/5.1.1.0/ITDStart.aspx?SVC=f3fd94ad-4a91-4962-b764-2d4caccal061

Exam Section 1: Item 16 o f 50 I Mark

N ational Board o f M edical E xam iners C om prehensive Basic Science Self-Assessment

T im e R em aining: 3 hr 31 m in 41 sec

16. A male newborn is delivered at 38 weeks' gestation following an uncomplicated pregnancy. Twelve-hours later, he begins sucking frantically and crying inconsolably. He overreacts to stimuli around him and has a marked startle response. The symptoms slowly resolve over the next 2 to 3 weeks. The most likely explanation for these symptoms is maternal use of which of the following substances during pregnancy? O A) Alcohol 0 B) Heroin C) LSD D)
0

Marijuana

E) Psilocybin

r -'

National Board o f Medical Examiners - Google Chrome https://www.starttest.com/api/5.1.1.0/ITDStart.aspx?SVC=f3fd94ad-4a91-4962-b764-2d4caccal061

Exam Section 1: Item 17 o f 50 I Mark

N ational Board o f M edical E xam iners C om prehensive Basic Science Self-Assessment

T im e R em aining: 3 hr 28 m in 23 sec

17. A 27-year-old woman comes to the physician because of a 2-year history of sporadic double vision and a 1-year history of intermittent numbness and tingling in her arms. The symptoms became more severe 5 days ago after a 4-day visit from her husband's parents. During the past year, she also has had four episodes of urinary incontinence. Physical examination shows mild dysarthria. On mental status examination, she has a mildly worried mood and a reactive affect. MRI of the brain shows a few small, scattered, nonspecific white-matter plaques. Which of the following is the most likely diagnosis? A) Complex partial seizure disorder

O B) Conversion disorder O C) Huntington disease o D) Multiple sclerosis E) Panic disorder without agoraphobia

N ationa: Board o f M ed ica l Examiners - G o o g le C hrom e

[ o

| 0

h ttp s://w w w .sta rtte st.e o m /a p i/5 .l.1.0/TTDStart.aspx?SVC=f3fd94ad-4a91-4962-b764-2d4caccal061

Exam Section 1: Item 18 o f 50 M ark

N ational Board o f M edical E xam iners C om prehensive Basic Science Self-Assessment

T im e R em aining: 3 hr 28 m in 14 sec

18. A 30-year-old woman, gravida 2, para 2, develops severe vaginal bleeding immediately after vaginal delivery of a male newborn at term. The hemorrhage continues during the next hour, and the patient undergoes an emergency hysterectomy to control the bleeding. Examination of the uterus shows fragments of placenta adherent to the uterine wall. There is absence of the decidua basalis. Which of the following is the most likely diagnosis? O A) Abruptio placentae O B) Chronic villitis C) Circumvallate placenta D) Placenta accreta O E) Succenturiate lobe

o o 31l ? $ Tpr F
Previous Next

Lab Values

Review

Help

Pause

C l
il

r[^ ,T|[

^4
^

1,5

11:37 AM 28/08/2012

r -'

National Board o f Medical Examiners - Google Chrome https://www.starttest.com/api/5.1.1.0/TTDStart.aspx?SVC=f3fd94ad-4a91-4962-b764-2d4caccal061

Exam Section 1: Item 19 o f 50 I Mark

N ational Board o f M edical E xam iners C om prehensive Basic Science Self-Assessment

T im e R em aining: 3 hr 27 m in 45 sec

19. A 55-year-old man with hepatic encephalopathy is brought to the physician because of confusion and increased lethargy during the past 24 hours. He has a history of cirrhosis of the liver caused by hepatitis C. He is easily arousable despite the lethargy. His temperature is 37C (98.6F). Physical examination shows scleral icterus, asterixis, a protuberant abdomen, and ascites. Test of the stool for occult blood is negative. Administration of which of the following drugs is most likely to improve this patient's mental status? O A) Bisacodyl O B) Docusate C) Lactulose 0 D) Octreotide 0; E) Omeprazole 0 F) Sucralfate 0 G) Ursodiol

o Previous

o Next $

@ Lab Values

Review

Help

Pause ^ ^ il

T p r

F 3 1 l

4
1,5

11:38 AM 28/08/2012

r -'

National Board o f Medical Examiners - Google Chrome https://www.starttest.com/api/5.1.1.0/ITDStart.aspx?SVC=f3fd94ad-4a91-4962-b764-2d4caccal061

Exam Section 1: Item 20 o f 50 I Mark

N ational Board o f M edical E xam iners C om prehensive Basic Science Self-Assessment

T im e R em aining: 3 hr 27 m in 17 sec

20. A 76-year-old man is admitted to the hospital because of a 2-week history of a persistent urinary tract infection. He is 165 cm (5 ft 5 in) and weighs 55 kg (121 lb); BMI is 20 kg/m2. The physician plans to initiate treatment. A component of the initial treatment includes an intravenous bolus dose of an antibiotic to attain an initial peak serum concentration of 8 mg/L. The volume of distribution of the antibiotic is 0.35 L/kg. Laboratory studies show a 24-hour urine creatinine clearance within the reference range. Which of the following is the most appropriate loading dose of this drug for this patient (in mg)? A) 25 B) 50 C) 100 D ) 150 E )200

p 1
Previous Lab Values Review Pause 11:38 AM 28/08/2012

National Board o f Medical Examiners - Google Chrome Q https:/?www.starttest.com/api/5.1.1.0/TTDStart.aspx?SVC=f3fd94ad-4a91-4962-b764-2d4caccal061

Exam Section 1: Item 21 o f 50 I Mark

N ational Board o f M edical E xam iners C om prehensive Basic Science Self-Assessment

T im e R em aining: 3 hr 17 m in 39 sec

21. A healthy 22-year-old man is a subject in a study investigating the regulation of heart rate during exercise. He is sitting on a stationary bicycle and is instructed to begin pedaling in 5 minutes. One minute before he begins to pedal, his heart rate increases. An increase in which of the following transmitters acting at the indicated receptor best explains the increase in heart rate in this man?

Transmitter
A) B) C) D) E) Acetylcholine Acetylcholine Norepinephrine Norepinephrine Norepinephrine

Receptor
muscarinic nicotinic a pi P2

National Board o f Medical Examiners - Google Chrome Q https:/?www.starttest.com/api/5.1.1.0/TTDStart.aspx?SVC=f3fd94ad-4a91-4962-b764-2d4caccal061

Exam Section 1: Item 22 o f 50 I Mark

N ational Board o f M edical E xam iners C om prehensive Basic Science Self-Assessment

T im e R em aining: 3 hr 16 m in 12 sec

22.

A 15-year-old boy has intracranial neoplasms, mental retardation, and seizures associated with tuberous sclerosis complex, an autosomal dominant disorder. His 41-year-old mother is healthy but has angiofibromas, which are benign skin lesions characteristic of tuberous sclerosis complex. Which of the following best explains the findings in the boy and his mother? A) Alternate paternity B) Anticipation C) Parental germline mosaicism

O D) Reduced penetrance E) Variable expressivity

National Board o f Medical Examiners - Google Chrome Q https:/?www.starttest.com/api/5.1.1.0/TTDStart.aspx?SVC=f3fd94ad-4a91-4962-b764-2d4caccal061

Exam Section 1: Item 23 o f 50 I Mark

N ational Board o f M edical E xam iners C om prehensive Basic Science Self-Assessment

T im e R em aining: 3 hr 14 m in 45 sec

23. A 40-year-old woman is brought to the emergency department 30 minutes after sustaining severe chest injuries in a motor vehicle collision. Despite appropriate treatment, she dies 1 hour later. She had a 25-year history of a wellcontrolled seizure disorder. A photograph of a coronal section of the brain obtained at autopsy is shown. Which of the following is the most likely cause of this patient's seizure disorder? A) Glioblastoma multiforme B) Herpes simplex encephalitis C) Mesial temporal sclerosis D) Neonatal ischemic stroke E) Vascular malformation

o o re*reii ?
Previous Next

Lab Values

Review

Help

Pause

T iri,

*"

. *V

11:51 AM 28/08/2012

National Board o f Medical Examiners - Google Chrome Q https://w vvw .starttest.eom /api/5.l.1.0/TTDStart.aspx?SVC=f3fd94ad-4a91-4962-b764-2d4caccal061

Exam Section 1: Item 24 o f 50 H Mark

National Board o f Medical Examiners Comprehensive Basic Science Self-Assessment

Tim e Remaining: 3 hr 12 min 3 sec

24. A 51-year-old man comes to the physician because of a 2-week history of cough productive of blood. He also has had a 3-kg (6.6-lb) weight loss during the past month. He has smoked 2 packs of cigarettes daily for 32 years. He has hypertension treated with metoprolol. Physical examination shows clubbing of the fingernails. Wheezes are heard over the right upper lobe anteriorly; there are no crackles. Laboratory studies show: Serum Na+ K+ c i3 Urea nitrogen Creatinine Osmolality
hco

Urine osmolality

125 mEq/L 3.9 mEq/L 85 mEq/L 25 mEq/L 8 mg/dL 0.4 mg/dL 262 mOsmol/kg 564 mOsmol/kg

A CT scan of the chest shows a 2.4 x 1.2-cm, spiculated noncalcified mass in the right upper lobe eroding into the right bronchus. This patient's laboratory findings are most likely caused by an excess of which of the following hormones? O A) ADH (vasopressin) O B) Adrenocorticotropic hormone O C) Brain natriuretic peptide D) Cortisol O E) Thyroxine (T4)

Previous

Next

Lab Values

Review

Help

Pause

$ Tpr F31l

1 PS l^ ip r ; Hu

il

4
1,5

11:54 AM 28/08/2012

National Board o f Medical Examiners - Google Chrome Q https:/?www.starttest.com/api/5.1.1.0/TTDStart.aspx?SVC=f3fd94ad-4a91-4962-b764-2d4caccal061

Exam Section 1: Item 25 o f 50 I Mark

N ational Board o f M edical E xam iners C om prehensive Basic Science Self-Assessment

T im e R em aining: 3 hr 10 m in 48 sec

25. A 60-year-old man comes to the physician because of weakness and light-headedness for 3 months. His diet consists of a healthy balance of fruits, vegetables, whole grains, and lean chicken and fish. He has a 10-year history of chronic autoimmune (Hashimoto) thyroiditis treated with thyroxine. Physical examination shows pallor. Serum studies show increased concentrations of methylmalonic acid and total homocysteine. A peripheral blood smear shows megaloblastic anemia. Which of the following is the most likely cause of the findings in this patient? A) Cystathionine 3-synthase deficiency

O B) Failure of intrinsic factor production C) Folic acid deficiency D)

Helicobacter pylori gastritis

E) Methylmalonyl-CoA mutase deficiency

National Board o f Medical Examiners - Google Chrome Q h ttp s://w w w .sta rtte st.e o m /a p i/5 .l.1.0/TTDStart.aspx?SVC=f3fd94ad-4a91-4962-b764-2d4caccal061

Exam Section 1: Item 26 o f 50 H Mark

National Board o f Medical Examiners Comprehensive Basic Science Self-Assessment

Tim e Remaining: 3 hr 9 min 28 sec

26.

A 55-year-old woman is scheduled to undergo transvaginal hysterectomy and oophorectomy for dysfunctional uterine bleeding. During the procedure, the uterus must be separated from all surrounding pelvic structures. Identification and incision of which of the following structures that attaches to the cervical region and extends posteriorly is most appropriate in this patient?

C A) Mesometrium B) D) Mesosalpinx Ovarian ligament C C) Mesovarium C E) Round ligament of the uterus F) Uterosacral ligament

Previous

o $ Tpr F31l
Next

Lab Values

Review

Help

Pause

r[^ ,T|[

il

^4
^

1,5

11:56 AM 28/08/2012

National Board o f Medical Examiners - Google Chrome Q h ttp s://w w w .sta rtte st.e o m /a p i/5 .l.1.0/TTDStart.aspx?SVC=f3fd94ad-4a91-4962-b764-2d4caccal061

Exam Section 1: Item 27 o f 50 H Mark

National Board o f Medical Examiners Comprehensive Basic Science Self-Assessment

Tim e Remaining: 3 hr 8 min 51 sec

27.

A 33-year-old woman comes to the physician because of a 16-kg (35-lb) weight gain during the past 6 months despite dieting. She is 168 cm (5 ft 6 in) tall and now weighs 104 kg (230 lb); BMI is 37 kg/m2. Her blood pressure is 156/96 mm Hg. Physical examination shows a round, ruddy face with hirsutism, excess weight predominantly in the shoulders and abdomen, and purple striae over the abdomen. An MRI shows a mass in the pituitary gland. A CT scan of the abdomen shows enlarged adrenal glands. Which of the following histologic changes would be expected in this patient's adrenal glands? A) Cortical hyperplasia

C B) Cortical hypertrophy C) Cortical metaplasia D) Medullary hyperplasia E) Medullary hypertrophy F) Medullary metaplasia

O o 31 ? $ Tpr F
Previous Next l

Lab Values

Review

Help

Pause

il

Hu

^4
-a

1,5

11:57 AM 28/08/2012

National Board o f Medical Examiners - Google Chrome Q https:/?www.starttest.com/api/5.1.1.0/TTDStart.aspx?SVC=f3fd94ad-4a91-4962-b764-2d4caccal061

Exam Section 1: Item 28 o f 50 I Mark

N ational Board o f M edical E xam iners C om prehensive Basic Science Self-Assessment

T im e R em aining: 3 hr 6 m in 43 sec

28.

A 22-year-old woman comes to the physician for a follow-up examination. One year ago, she was diagnosed with a pulmonary embolism. Two years ago, she delivered a female stillborn at 23 weeks' gestation. Physical examination today shows no abnormalities. Laboratory studies show a platelet count of 250,000/mm3, a prothrombin time within the reference range, and an increased partial thromboplastin time. The findings in this patient are most consistent with which of the following conditions? A) C) Antiphospholipid antibody syndrome Increased factor VIII (antihemophilic factor) concentration

O B) Factor V Leiden mutation D) Protein C deficiency E) Prothrombin G20210A mutation

National Board o f Medical Examiners - Google Chrome Q https:/?www.starttest.com/api/5.1.1.0/TTDStart.aspx?SVC=f3fd94ad-4a91-4962-b764-2d4caccal061

Exam Section 1: Item 29 o f 50 I Mark

N ational Board o f M edical E xam iners C om prehensive Basic Science Self-Assessment

T im e R em aining: 3 hr 5 m in 17 sec

29. A 54-year-old man comes to the physician because of a 2-week history of burning epigastric pain. He is recently divorced. His diet mainly consists of fast food. He smokes cigars and drinks one to two 12-ounce cans of beer on the weekends. He was treated for Helicobacter pylori infection 1 year ago. He uses high doses (600 mg daily) of over-the-counter ibuprofen for chronic joint pain. Abdominal examination shows tenderness to deep palpation of the epigastric area. A peptic ulcer is suspected. Which of the following is the strongest predisposing risk factor for this patient's condition? O A) High-fat diet O B) Ibuprofen use O C) Ingestion of alcohol D) Recent psychosocial stressor O E) Smoking cigars

National Board o f Medical Examiners - Google Chrome Q https:/?www.starttest.com/api/5.1.1.0/TTDStart.aspx?SVC=f3fd94ad-4a91-4962-b764-2d4caccal061

Exam Section 1: Item 30 o f 50 I Mark

N ational Board o f M edical E xam iners C om prehensive Basic Science Self-Assessment

T im e R em aining: 3 hr 4 m in 11 sec

30. A 22-year-old nulligravid woman comes to the physician because she has been unable to conceive for 18 months. She has had irregular menses since menarche at the age of 13 years. She is 165 cm (5 ft 5 in) tall and weighs 88 kg (195 lb); BMI is 32 kg/m2. Her temperature is 36.8C (98.2F), pulse is 88/min, respirations are 14/min, and blood pressure is 140/90 mm Hg. Physical examination shows moderate hirsutism and mild acne over the face and back. Pelvic examination shows no abnormalities of the exterior genitalia; a bimanual examination is limited by the patient's obesity. Which of the following is the most likely cause of this patient's infertility? A) Hypothyroidism B) Klinefelter syndrome C) Pituitary adenoma D) Polycystic ovarian syndrome E) Primary ovarian failure

National Board o f Medical Examiners - Google Chrome Q https:/?www.starttest.com/api/5.1.1.0/TTDStart.aspx?SVC=f3fd94ad-4a91-4962-b764-2d4caccal061

Exam Section 1: Item 31 o f 50 I Mark

N ational Board o f M edical E xam iners C om prehensive Basic Science Self-Assessment

T im e R em aining: 3 hr 3 m in 14 sec

31. A 9-year-old girl is brought to the emergency department by her father because of severe pain in her right shoulder after a fall 1 hour ago. Physical examination shows pain on movement of the right shoulder and a tender right clavicle. An x-ray of the shoulder shows a nondisplaced fracture of the right clavicle. Which of the following structures of the clavicle will most likely assist in producing new bone to heal this fracture? O A) Epiphysis B) Haversian canal C) Lamella D) Periosteum E) Volkmann canal

National Board o f Medical Examiners - Google Chrome Q h ttp s://w w w .sta rtte st.e o m /a p i/5 .l.1.0/TTDStart.aspx?SVC=f3fd94ad-4a91-4962-b764-2d4caccal061

Exam Section 1: Item 32 o f 50 H Mark

National Board o f Medical Examiners Comprehensive Basic Science Self-Assessment

Tim e Remaining: 3 hr 1 min 30 sec

32. A 2-month-old boy dies of meningitis despite appropriate antibiotic therapy. Culture of his cerebrospinal fluid grows mycobacterial infections. Which of the following immunodeficiencies most likely explains this familial history? C A) Complement deficiency C B) Interferon-Y receptor deficiency C C) Leukocyte adhesion deficiency D)X-linked agammaglobulinemia

Mycobacterium tuberculosis. Two of his brothers died of atypical

Previous

p 1
Lab Values

Review

Pause

C l
12:04 PM 28/08/2012

National Board o f Medical Examiners - Google Chrome Q https:/7www.starttest.com/api/5.1.1.0/ITDStart.aspx?SVC=f3fd94ad-4a91-4962-b764-2d4caccal061

Exam Section 1: Item 33 o f 50 I Mark

N ational Board o f M edical E xam iners C om prehensive Basic Science Self-Assessment

T im e R em aining: 3 hr 1 m in 15 sec

33. A previously healthy 25-year-old man is admitted to the hospital for treatment of viral pneumonia. He is mechanically ventilated because of respiratoryfailure due to hypoxemia. An openlung biopsy specimen is obtained and shows a pattern of acute lung injury. The presence of which of the following in the biopsy specimen most clearly suggests diffuse alveolar damage rather than bronchiolitis obliterans-organizing pneumonia in this patient? A) Alveolar hyaline membrane B) Foamy macrophages C) Interstitial collagen deposition O D) Masson bodies E) Uniform fibroblastic proliferation

National Board o f Medical Examiners - Google Chrome Q https:/7www.starttest.com/api/5.1.1.0/ITDStart.aspx?SVC=f3fd94ad-4a91-4962-b764-2d4caccal061

Exam Section 1: Item 34 o f 50 I Mark

N ational Board o f M edical E xam iners C om prehensive Basic Science Self-Assessment

T im e R em aining: 2 hr 59 m in 45 sec

34. A 39-year-old woman with end-stage renal disease caused by systemic lupus erythematosus undergoes kidney transplantation from a living, unrelated donor. Despite aggressive immunosuppressive therapy, the transplant is rejected after 6 months. Analysis of a biopsy specimen of the transplant shows numerous infiltrating CD8+ T lymphocytes. Which of the following is the most likely cause of rejection of the transplant in this patient? O A) Antibody synthesis O B) Complement activation O C) Direct cytotoxicity D) Histamine release O E) Nitric oxide production

National Board o f Medical Examiners - Google Chrome Q https:/?www.starttest.com/api/5.1.1.0/TTDStart.aspx?SVC=f3fd94ad-4a91-4962-b764-2d4caccal061

Exam Section 1: Item 35 o f 50 I Mark

N ational Board o f M edical E xam iners C om prehensive Basic Science Self-Assessment

T im e R em aining: 2 hr 58 m in 11 sec

35. A 7-year-old girl is brought to the physician by her parents because of a 3-year history of temper tantrums when her parents leave her. She refuses to sleep alone and recently would not attend her friend's birthday slumber party. She calls to her mother if she is in another room of the house. Each morning, the patient's teacher has to carry her into school crying. Her mother recently returned to work full-time and says that she is quite angry that her daughter is behaving in such a defiant way. Her father recalls that he behaved the same way when he was her age. Which of the following is the most likely explanation for this patient's behavior? O A) Conduct disorder B) Oppositional defiant disorder C) Post-traumatic stress disorder D) Separation anxiety disorder O E) Normal behavior

National Board o f Medical Examiners - Google Chrome Q https:/?www.starttest.com/api/5.1.1.0/TTDStart.aspx?SVC=f3fd94ad-4a91-4962-b764-2d4caccal061

Exam Section 1: Item 36 o f 50 I Mark

N ational Board o f M edical E xam iners C om prehensive Basic Science Self-Assessment

T im e R em aining: 2 hr 56 m in 25 sec

36. A 49-year-old man with pancreatic cancer undergoes a Whipple procedure to resect the distal stomach, duodenum, and head of the pancreas. During the procedure, the cut end of the remaining stomach is joined to the side of the jejunum (gastrojejunostomy). Which of the following nerve structures can be cut to decrease stomach acid output and decrease the likelihood of ulcers forming in the jejunum at the site of junction with the stomach? O A) Celiac plexus B) Iliohypogastric nerves C) Phrenic nerves O D) Subcostal nerves O E) Sympathetic chain O F) Vagal trunks

National Board o f Medical Examiners - Google Chrome Q https:/?www.starttest.com/api/5.1.1.0/TTDStart.aspx?SVC=f3fd94ad-4a91-4962-b764-2d4caccal061

Exam Section 1: Item 37 o f 50 I Mark

N ational Board o f M edical E xam iners C om prehensive Basic Science Self-Assessment

T im e R em aining: 2 hr 56 m in 9 sec

37. A 35-year-old man comes to the physician for a routine examination. He has a 5-year history of recurrent episodes of genital herpes treated with acyclovir. He says, "I've been taking the medication, but I still occasionally get lesions. When will I be cured?" The physician informs him that herpes simplex is a lifelong infection, with no cure at the present time. Which of the following best explains the longevity of the herpes simplex virus? A) Continual replication in dendritic cells B) Continual replication in epithelial cells of the skin C) Continual replication in sacral root ganglia D) Continual replication in T lymphocytes E) Establishment of a latent infection in B lymphocytes F) Establishment of a latent infection in circulating immune cells G) Establishment of a latent infection in epithelial cells of the skin H) Establishment of a latent infection in sensory nerve cells

National Board o f Medical Examiners - Google Chrome Q https:/?www.starttest.com/api/5.1.1.0/TTDStart.aspx?SVC=f3fd94ad-4a91-4962-b764-2d4caccal061

Exam Section 1: Item 38 o f 50 I Mark

N ational Board o f M edical E xam iners C om prehensive Basic Science Self-Assessment

T im e R em aining: 2 hr 52 m in 57 sec

38.

A 3-year-old boy is brought to the emergency department because of multiple ecchymoses, bleeding into the joints, and bloody diarrhea 48 hours after ingesting an unknown quantity of warfarin that he took from his grandmother's purse. Prothrombin time is 80 sec (INR=20). Which of the following is the most appropriate initial therapy? A) C) Factor IX (plasma thromboplastin component) concentrate Fresh frozen plasma

O B) Fibrinogen concentrate O D) Platelet concentrates E) Vitamin B 12(cyanocobalamin), parenterally

National Board o f Medical Examiners - Google Chrome Q https:/?www.starttest.com/api/5.1.1.0/TTDStart.aspx?SVC=f3fd94ad-4a91-4962-b764-2d4caccal061

Exam Section 1: Item 39 o f 50 I Mark

N ational Board o f M edical E xam iners C om prehensive Basic Science Self-Assessment

T im e R em aining: 2 hr 52 m in 4 sec

39. An 18-year-old man comes to the physician 6 hours after he twisted his ankle while playing football. Physical examination shows looseness of the lateral collateral ligament, suggestive of a tear. Lack of which of the following components in the ligament will most likely limit healing of this injury? O A) Blood vessels O B) Collagen fibers O C) Elastic fibers O D) Lymphatic vessels E) Nerves

National Board o f Medical Examiners - Google Chrome Q https:/?www.starttest.com/api/5.1.1.0/TTDStart.aspx?SVC=f3fd94ad-4a91-4962-b764-2d4caccal061

Exam Section 1: Item 40 o f 50 I Mark

N ational Board o f M edical E xam iners C om prehensive Basic Science Self-Assessment

T im e R em aining: 2 hr 51 m in 17 sec

40.

A previously healthy 6-year-old girl is brought to the emergency department because of severe abdominal pain, nausea, and vomiting that began 4 hours after attending a friend's birthday party. Cake and other pastries were served at the party. She appears acutely ill and mildly dehydrated. Her temperature is 37C (98.6F), pulse is 104/min, respirations are 20/min, and blood pressure is 96/60 mm Hg. Examination shows no abnormalities of the skin. Bowel sounds are hyperactive. Which of the following is the most likely cause of these findings?

O A) Bacteremia B) Enterotoxin ingestion C) Intestinal mucosa inflammation O D) Lead toxicity E) Villus atrophy F)Virem ia

National Board o f Medical Examiners - Google Chrome Q https://w vvw .starttest.eom /api/5.l.1.0/TTDStart.aspx?SVC=f3fd94ad-4a91-4962-b764-2d4caccal061

Exam Section 1: Item 41 o f 50 H Mark

National Board o f Medical Examiners Comprehensive Basic Science Self-Assessment

Tim e Remaining: 2 hr 50 min 2 sec

41. A 28-year-old woman comes to the physician for a health maintenance examination. During the interview, she tearfully tells the physician that she and her new husband are having problems. She says she wants to go out with friends, but he does not enjoy being with people, preferring individual activities such as hiking. She says he seems indifferent to sexual intimacy and neither shows much emotion nor understands her feelings at all. Which of the following personality styles best explains the husband's behavior? A) Avoidant C B) Narcissistic C) Paranoid D) Schizoid C E) Schizotypal

O o 31 ? $ Tpr F
Previous Next l

Lab Values

Review

Help

Pause

Hu

il

^4
^

1,5

12:16 PM 28/08/2012

National Beard of Medical Examiners - Google Chrome


Q h ttps://w w w .startte st.e om /a p i/5 .l.1.0/TTDStart.aspx?SVC=f3fd94ad-4a91-4962-b764-2d4caccal061

I <=> I

3 *|

Exam Section 1: Item 42 o f 50 H Mark

National Board o f Medical Examiners Comprehensive Basic Science Self-Assessment

Tim e Remaining: 2 hr 48 min 16 sec

42. A 23-year-old woman is treated with acetazolamide to prevent acute altitude sickness 3 days before going on a mountain-climbing expedition. Acetazolamide acts predominately in which of the following areas of the kidney in this patient? O A) Ascending loop of Henle O B) Collecting ducts O C) Descending loop of Henle D) Distal tubule E) Proximal tubule

Previous

Lab Values

Review

Help

Pause

O
* ..ill ><>

fTi

12:18 PM 28/08/2012

National Board o f Medical Examiners - Google Chrome Q https:/?www.starttest.com/api/5.1.1.0/TTDStart.aspx?SVC=f3fd94ad-4a91-4962-b764-2d4caccal061

Exam Section 1: Item 43 o f 50 I Mark

N ational Board o f M edical E xam iners C om prehensive Basic Science Self-Assessment

T im e R em aining: 2 hr 47 m in 47 sec

43.

A study is conducted to assess the effectiveness of vitamin C in preventing common colds in children. Patients are randomly assigned to receive either vitamin C or no intervention. Outcome events (common cold) are reported by the parents on a monthly basis for 1 year. Results show that those taking vitamin C have fewer colds compared with those who received no intervention (p<0.05). Which of the following raises the most concern about the validity of the conclusion that vitamin C supplements prevent the common cold? A) Inadequate statistical power

O B) Nonrandomization O C) Selection bias D) Variability in outcome assessment

National Beard of Medical Examiners - Google Chrome


Q h ttps://w w w .startte st.e om /a p i/5 .l.1.0/TTDStart.aspx?SVC=f3fd94ad-4a91-4962-b764-2d4caccal061

I <=> I

3 *|

Exam Section 1: Item 44 o f 50 H Mark

National Board o f Medical Examiners Comprehensive Basic Science Self-Assessment

Tim e Remaining: 2 hr 45 min 36 sec

44. A healthy 21-year-old man dies suddenly of cardiac arrest after snorting cocaine. Cocaine-induced inhibition of which of the following is the most likely explanation for the cardiac arrest? A) Acetylcholinesterase

O B) a2 -Adrenergic receptors O C) (32 -Adrenergic receptors O D) Monoamine oxidase O E) Muscarinic receptors O F) Norepinephrine reuptake

Previous

Lab Values

Review

Help

Pause

O
12:21 PM 28/08/2012

fTi

National Board o f Medical Examiners - Google Chrome Q https:/?www.starttest.com/api/5.1.1.0/TTDStart.aspx?SVC=f3fd94ad-4a91-4962-b764-2d4caccal061

Exam Section 1: Item 45 o f 50 I Mark

N ational Board o f M edical E xam iners C om prehensive Basic Science Self-Assessment

T im e R em aining: 2 hr 44 m in 48 sec

45. A 62-year-old man is brought to the emergency department because of a 3-hour history of progressive difficulty breathing and mild left shoulder pain. His symptoms began after he ran up several flights of stairs. He has poorly controlled hypertension and mild angina pectoris. He has smoked one-half pack of cigarettes daily for 40 years. He appears uncomfortable and has labored breathing. Diffuse crackles, rhonchi, and scattered wheezing are heard on auscultation of the posterior lung fields. His arterial Po2 is 58 mm Hg. Which of the following is the most likely diagnosis? A) Cardiac tamponade

O B) Pneumonitis O C) Pneumothorax O D) Pulmonary edema O E) Pulmonary embolism

Previous

Next

&
Lab Values Review Help

Pause

C l
...i,

< >

12:22 PM

National Board o f Medical Examiners - Google Chrome Q https:/?www.starttest.com/api/5.1.1.0/TTDStart.aspx?SVC=f3fd94ad-4a91-4962-b764-2d4caccal061

Exam Section 1: Item 46 o f 50 I Mark

N ational Board o f M edical E xam iners C om prehensive Basic Science Self-Assessment

T im e R em aining: 2 hr 43 m in 19 sec

46. A 22-year-old woman is admitted to the hospital because of a 10-day history of polydipsia and polyuria. She says that the urge to urinate often awakens her at night. She has been taking lithium carbonate for 2 years for bipolar disorder; her dosage was increased 6 months ago because of recurrent severe manic episodes. Her vital signs are within normal limits. Physical examination shows no abnormalities. Over the next 24 hours, urine excretion totals 6.5 L. Laboratory studies at this time show a serum sodium concentration of 148 mEq/L, serum osmolality of 315 mOsmol/kg, and urine osmolality of 75 mOsmol/kg. After administration of desmopressin, urine output and osmolality do not change. Which of the following findings in the nephron best describes the tubular osmolality, compared with serum, in this patient?

Proximal Tubule
A) B) C) D) E) F) G) H) I) Hypertonic Hypertonic Hypertonic Isotonic Isotonic Isotonic Hypotonic Hypotonic Hypotonic

Juxtaglomerular Apparatus
hypertonic hypertonic hypotonic isotonic hypotonic hypotonic hypertonic hypotonic hypotonic

Medullary Collecting Duct


hypertonic hypotonic hypotonic isotonic hypertonic hypotonic hypertonic hypertonic hypotonic

O Previous

o Next $ i m i

@ Lab Values

Review

Help

Pause

t f "

.iii ffip 4 > 28/08/2012 i


-a 12:23 PM

National Board o f Medical Examiners - Google Chrome Q https:/?www.starttest.com/api/5.1.1.0/TTDStart.aspx?SVC=f3fd94ad-4a91-4962-b764-2d4caccal061

Exam Section 1: Item 47 o f 50 I Mark

N ational Board o f M edical E xam iners C om prehensive Basic Science Self-Assessment

T im e R em aining: 2 hr 40 m in 22 sec

47. An 82-year-old woman is admitted to the hospital for treatment of a smallright pleural effusion.Physical examination showsdullness to percussion at thelevel of the 7th rib and below on the right. A thoracentesis is done. The patient is sitting up, supportedby leaning against abedside table. After preparing the skin and deliveringa local anesthetic,into which of the following locations on the right is it most appropriate to insert the catheter? O A) Above the 5th rib in the midscapular line B) Above the 5th rib just to the right of the sternum C) Above the 7th rib in the paravertebral region D) Above the 9th rib in the midscapular line O E) Below the 5th rib in the midaxillary line F) Below the 5th rib in the midclavicular line G) Below the 9th rib in the midscapular line

National Board o f Medical Examiners - Google Chrome Q https:/?www.starttest.com/api/5.1.1.0/TTDStart.aspx?SVC=f3fd94ad-4a91-4962-b764-2d4caccal061

Exam Section 1: Item 48 o f 50 I Mark

N ational Board o f M edical E xam iners C om prehensive Basic Science Self-Assessment

T im e R em aining: 2 hr 38 m in 5 sec

48. A 72-year-old woman is admitted to the intensive care unit after undergoing immediate operative repair of a ruptured colonic diverticulum. During the next 2 weeks, she develops recurrent fevers. After a blood culture grows yeast, micafungin therapy is initiated. This drug most likely targets which of the following cellular structures? A) Cell wall B) Endospore C) Kinetoplast D) Ribosome O E) Transcription complex

National Board o f Medical Examiners - Google Chrome Q https:/?www.starttest.com/api/5.1.1.0/TTDStart.aspx?SVC=f3fd94ad-4a91-4962-b764-2d4caccal061

Exam Section 1: Item 49 o f 50 I Mark

N ational Board o f M edical E xam iners C om prehensive Basic Science Self-Assessment

T im e R em aining: 2 hr 37 m in 0 sec

49. A 23-year-old woman comes to the physician because of a 1-day history of chills, nausea, and vomiting. She also has a 4-day history of flank pain, urinary frequency and urgency, and pain with urination. Her temperature is 38.9C (102F), pulse is 125/min, and blood pressure is 96/60 mm Hg. Physical examination shows right costovertebral angle tenderness. Urinalysis shows 25 WBC/hpf. A urine sample grows pink colonies on lactose-MacConkey agar, produces a characteristic green sheen on eosin-methylene blue agar, and is indole positive. The production of which of the following by the bacterium aids in the virulence of the causal organism in this patient? O A)Adhesins B) Anti hemolytic factors O C) Pyrogenic exotoxin O D) Teichoic acid in the cell wall O E) Thick peptidoglycan layer

National Board o f Medical Examiners - Google Chrome Q h ttps://w w w .sta rttest.eom /a p i/5 .l.1.0/TTDStart.aspx?SVC=f3fd94ad-4a91-4962-b764-2d4caccal061

Exam Section 1: Item 50 o f 50 H Mark

National Board o f Medical Examiners Comprehensive Basic Science Self-Assessment

Tim e Remaining:

50. A 38-year-old woman at 32 weeks' gestation is brought to the physician because of a 2-day history of fever, nausea, vomiting, and muscle aches. She appears acutely ill. Her temperature is 39C (102.2F), pulse is 120/min, and respirations are 20/min. A photomicrograph of a Gram stain of an organism recovered from a blood culture is shown. Which of the following is the most likely causal organism?

Corynebacterium urealyticum B) Gardnerella vaginalis C) Listeria monocytogenes D ) Rhodococcus equi E) Staphylococcus aureus
A)

r -'

National Board o f Medical Examiners - Google Chrome https://www.starttest.com/api/5.1.1.0/ITDStart.aspx?SVC=f3fd94ad-4a91-4962-b764-2d4caccal061

Exam Section 2: Item 1 o f 50 I Mark

N ational Board o f M edical E xam iners C om prehensive Basic Science Self-Assessment

T im e R em aining: 4 hr 19 m in 55 sec

1. A 21-year-old woman who is a medical student comes to the physician for evaluation of recurrent throbbing headache associated with sensitivity to light and sound. Neurologic examination shows no abnormalities. Sumatriptan is initiated for suspected migraine. This drug is most appropriate for this patient because of its ability to activate which of the following types of receptors? O A) Acetylcholine B) Y-Aminobutyric acid O C) Dopamine O D) Neurotensin 0 E) Serotonin

r -'

National Board o f Medical Examiners - Google Chrome https://www.starttest.com/api/5.1.1.0/ITDStart.aspx?SVC=f3fd94ad-4a91-4962-b764-2d4caccal061

Exam Section 2: Item 2 o f 50 I Mark

N ational Board o f M edical E xam iners C om prehensive Basic Science Self-Assessment

T im e R em aining: 4 hr 19 m in 23 sec

2. A 5-year-old girl is brought to the physician because of listlessness, fatigue, and dull pain in the right upper quadrant of the abdomen. Her height and weight are below the 25th percentile. Laboratory findings indicate that the content of her (3-globin chain is 15% to 20% of normal. Sequencing of the (3-globin gene shows a point mutation in a sequence 3' to the coding region in which AATAAA is converted to AACAAA. Consequently, the amount of mRNAfor (3-globin is decreased to 10% of normal. Which of the following functions in mRNA synthesis and processing is most likely encoded by the sequence AATAAA? A) Capping with GTP B) D) Cleavage and polyadenylation Splicing of the initial mRNA transcript in the nucleus C) Silencing of the promoter E) Transport of the mRNA out of the nucleus

o
Previous

o
Next

m
Lab Values

Review Help

to
Pause -a ^ il

~ m

' V

4
1,5

12:50 PM 28/08/2012

r -'

National Board o f Medical Examiners - Google Chrome https://www.starttest.com/api/5.1.1.0/ITDStart.aspx?SVC=f3fd94ad-4a91-4962-b764-2d4caccal061

Exam Section 2: Item 3 o f 50 I Mark

N ational Board o f M edical E xam iners C om prehensive Basic Science Self-Assessment

T im e R em aining: 4 hr 18 m in 54 sec

3. An investigator breeds a transgenic strain of mice that develops severe fasting hypoglycemia. Administration of glucagon does not correct the hypoglycemia, but administration of epinephrine results in an increase in the serum glucose concentration. Which of the following proteins is most likely defective in this mouse strain? O A) Adenylyl cyclase O B) Glucagon receptor C) E) Glycogen phosphorylase Phosphorylase kinase O D) Heterotrimeric G protein F) Protein kinase A

o
Previous

o
Next

m
Lab Values

Review

* *
Help

to
Pause -a ^ il

~ m

' V

4
1,5

12:50 PM 28/08/2012

r -'

National Board o f Medical Examiners - Google Chrome https://www.starttest.com/api/5.1.1.0/ITDStart.aspx?SVC=f3fd94ad-4a91-4962-b764-2d4caccal061

Exam Section 2: Item 4 o f 50 0 M ark

N ational Board o f M edical E xam iners C om prehensive Basic Science Self-Assessment

T im e R em aining: 4 hr 17 m in 35 sec

4. A 60-year-old man comes to the physician because of progressive weakness of his hands during the past 6 months. He works as a carpenter and has had difficulty handling his tools. He has smoked 1 pack of cigarettes daily for 45 years. He drinks two beers daily during the workweek and approximately six beers daily on weekends. Physical examination shows no lymphadenopathy. There is decreased strength (4/5) in the upper and lower extremities and atrophy of the muscles of the hands. He has diffuse hyperreflexia. Fasciculations are noticed on the muscles of the hands and upper extremities. Sensory examination is normal. Over the course of this disease, which of the following findings is most likely? O A) Dementia o B) Difficulty swallowing O C) Loss of facial sensation D) Loss of peripheral vibratory sensation O E) Nystagmus

o
Previous

o
Next

m
Lab Values

Review

p
Help

to
Pause -a ^ il

4
1,5

~ m

r "5

' V

l l ' Z

r @

12:51 PM 28/08/2012

r -'

National Board o f Medical Examiners - Google Chrome https://www.starttest.com/api/5.1.1.0/ITDStart.aspx?SVC=f3fd94ad-4a91-4962-b764-2d4caccal061

Exam Section 2: Item 5 o f 50 I Mark

N ational Board o f M edical E xam iners C om prehensive Basic Science Self-Assessment

T im e R em aining: 4 hr 17 m in 27 sec

5. A 60-year-old man has had episodes of fluctuating weakness for the past 2 years. The weakness was first noticed when he had difficulty lifting books. At times, he has had drooping of the eyelids and double vision. Recently, he has had difficulty climbing stairs and regurgitation of liquids into his nose. On physical examination, the weakness increases with repeated exercise. An x-ray of the chest shows a mass in the mediastinum. Which of the following laboratory assays is most likely to be positive? O A) Acetylcholine receptor antibody B) D) Botulinum toxin Rapid plasma reagin C) C-reactive protein E) Rheumatoid factor

o
Previous

o
Next

m
Lab Values

Review Help

to
Pause -a ^ il

~ m

' V

4
1,5

12:52 PM 28/08/2012

N ationa: Board o f M ed ica l Examiners - G o o g le C hrom e

[ o

| 0

h ttp s://w w w .sta rtte st.e o m /a p i/5 .l.1.0/TTDStart.aspx?SVC=f3fd94ad-4a91-4962-b764-2d4caccal061

Exam Section 2: Item 6 o f 50 M ark

N ational Board o f M edical E xam iners C om prehensive Basic Science Self-Assessment

T im e R em aining: 4 hr 16 m in 33 sec

6. A 3-year-old girl is brought to the emergency department by her mother 1 hour after she was found with a half-empty bottle of her grandmother's diabetes medication. The mother tells the physician that the child consumed approximately 25 metformin tablets. Physical examination shows no abnormalities. This patient is at greatest risk for which of the following serum abnormalities? A) Decreased calcium concentration B) Decreased glucose concentration C) Decreased sodium concentration O D) Increased AST and ALT activities E) Increased creatinine concentration F) Increased lactic acid concentration

o
Previous

o
Next

m
Lab Values

Review Help

to
Pause -a ^ il

~ m

' V

4
1,5

12:53 PM 28/08/2012

r -'

National Board o f Medical Examiners - Google Chrome https://www.starttest.com/api/5.1.1.0/ITDStart.aspx?SVC=f3fd94ad-4a91-4962-b764-2d4caccal061

Exam Section 2: Item 7 o f 50 I Mark

N ational Board o f M edical E xam iners C om prehensive Basic Science Self-Assessment

T im e R em aining: 4 hr 16 m in 8 sec

7. A 65-year-old woman with diabetic ketoacidosis has had a fever and proptosis of the left eye over the past 3 days. She is unable to move the left eye. A black eschar is present on the nasal mucosa. Which of the following is the most likely diagnosis? A) Aspergillosis B) Candidiasis O C) Coccidioidomycosis O D) Cryptococcosis E) Histoplasmosis O F) Mucormycosis

o
Previous

o
Next

m
Lab Values

Review

^
Help

to
Pause -a ^ il

~ m

' V

4
1,5

12:53 PM 28/08/2012

r -'

National Board o f Medical Examiners - Google Chrome https://www.starttest.com/api/5.1.1.0/ITDStart.aspx?SVC=f3fd94ad-4a91-4962-b764-2d4caccal061

Exam Section 2: Item 8 o f 50 I Mark

N ational Board o f M edical E xam iners C om prehensive Basic Science Self-Assessment

T im e R em aining: 4 hr 15 m in 40 sec

8. A 42-year-old man comes to the physician because of a 4-week history of muscle cramping and pain. Two months ago, he began treatment with simvastatin (80 mg daily) for hypercholesterolemia. After 1 month, marked improvement was noted in his serum LDL-cholesterol concentration, but serum triglyceride concentration remained increased. At that time, gemfibrozil was added to his regimen to decrease his triglyceride concentration. Physical examination today shows no abnormalities. This patient's myalgia is most likely related to which of the following effects of gemfibrozil on simvastatin? A) Decreased bioavailability B) Increased absorption O C) Inhibition of cytochrome P450 metabolism D) Inhibition of hepatic glycosylation E) Inhibition of hepatic sulfation

o
Previous

o
Next

m
Lab Values

Review Help

to
Pause -a ^ il

~ m

' V

4
1,5

12:54 PM 28/08/2012

N ationa: Board o f M ed ica l Examiners - G o o g le C hrom e

[ o

| 0

Q h ttps://w vvw .starttest.eom /api/5.l.1.0/ITDStart.aspx?SVC=f3fd94ad-4a91-4962-b764-2d4caccal061

Exam Section 2: Item 9 o f 50 |

National Board o f Medical Examiners Comprehensive Basic Science Self-Assessment

Tim e Remaining: 4 hr 12 min 32 sec

9. A 25-year-old woman comes to the physician because of a 2-day history of decreasing urine output despite ample fluid intake. She underwent operative removal of a perforated appendix 3 weeks ago. She received intravenous gentamicin for 2 weeks. Physical examination shows no abnormalities. Laboratory studies show: Serum Urea nitrogen Creatinine Urine Specific gravity Protein Na+ Casts 35 mg/dL 2.9 mg/dL 1.010 2+ 25 mEq/L brown granular

Histologic examination of this patient's kidneys would most likely show necrotic cells in which of the following components of the nephron? C A) Bowman capsule 0 B) Collecting tubule C) Glomerular basement membrane D) Mesangium 0 E) Proximal tubule

O
Previous

o
Next

m
Lab Values

Review

* *
Help

to
Pause ^ ^ il

4
1,5

~ m

' V

12:57 PM 28/08/2012

r -'

National Board o f Medical Examiners - Google Chrome https://www.starttest.com/api/5.1.1.0/ITDStart.aspx?SVC=f3fd94ad-4a91-4962-b764-2d4caccal061

Exam Section 2: Item 10 o f 50 I Mark

N ational Board o f M edical E xam iners C om prehensive Basic Science Self-Assessment

T im e R em aining: 4 hr 9 m in 48 sec

10. A 48-year-old man comes to the physician because he is concerned about his weight. He is 178 cm (5 ft 10 in) tall and weighs 91 kg (200 lb); BMI is 29 kg/m2. His waist circumference is 103 cm (41 in). His blood pressure is 140/90 mm Hg. Physical examination shows a protuberant abdomen. Laboratory studies show fasting serum glucose and insulin concentrations that are increased and remain increased 2 hours after the patient receives 75 g of oral glucose. Which of the following sets of additional laboratory findings in serum is most likely in this patient?

Triglycerides
A) B) C) D) E) F) * * *

HDL-cholesterol

Free Fatty Acids

i i i

i + i i

i + i + i

O
Previous

o
Next

m
Lab Values

Review Help

to
Pause -a ^ il

~ m

' V

4
1,5

12:59 PM 28/08/2012

N ationa: Board o f M ed ica l Examiners - G o o g le C hrom e

[ o

| 0

Q h ttp s://w w w .sta rtte st.e o m /a p i/5 .l.1.0/TTDStart.aspx?SVC=f3fd94ad-4a91-4962-b764-2d4caccal061

Exam Section 2: Item 11 o f 50 | l^ ark

National Board o f Medical Examiners Comprehensive Basic Science Self-Assessment

Tim e Remaining: 4 hr 7 min 35 sec

11. A 47-year-old man is brought to the emergency department 3 hours after the sudden onset of sharp left flank pain and fever. He has had similar episodes during the past 5 years. His temperature is 38.3C (100.9F). Abdominal examination shows exquisite tenderness in the left costovertebral region. Laboratory studies show: Hemoglobin Hematocrit Leukocyte count Segmented neutrophils Bands Lymphocytes Monocytes Urine leukocytes 16 g/dL 48% 18,000/mm3 85% 5% 8%

2%

2500/mm3

Imaging studies show hydronephrosis of the left kidney and a staghorn calculus. A nephrectomy is done; the calculus is composed of magnesium ammonium phosphate (struvite). Culture of a urine sample is most likely to grow which of the following organisms? C A)

Escherichia coli B) Proteus mirabilis C) Pseudomonas aeruginosa D) Serratia marcescens E) Staphylococcus aureus

r -'

National Board o f Medical Examiners - Google Chrome https://www.starttest.com/api/5.1.1.0/TTDStart.aspx?SVC=f3fd94ad-4a91-4962-b764-2d4caccal061

Exam Section 2: Item 12 o f 50 I Mark

N ational Board o f M edical E xam iners C om prehensive Basic Science Self-Assessment

T im e R em aining: 4 hr 7 m in 2 sec

12. A 43-year-old man comes to the physician for a routine health maintenance examination. He is 170 cm (5 ft 7 in) tall and weighs 86 kg (188 lb); BMI is 30 kg/m2. Physical examination and laboratory studies show no other abnormalities. He tells the physician, "My older brother just got diagnosed with diabetes. I don't want that to happen to me. What should I do?" Which of the following diets is most likely to be effective in decreasing this patient's risk for type 2 diabetes mellitus?
0 0 0 0 0

A) Low-calorie B) Low-carbohydrate C) Low-cholesterol D) Low-protein E) Low-sodium

r -'

National Board o f Medical Examiners - Google Chrome https://www.starttest.com/api/5.1.1.0/ITDStart.aspx?SVC=f3fd94ad-4a91-4962-b764-2d4caccal061

Exam Section 2: Item 13 o f 50 I Mark

N ational Board o f M edical E xam iners C om prehensive Basic Science Self-Assessment

T im e R em aining: 4 hr 6 m in 30 sec

13. An 18-year-old primigravid woman at approximately 16 weeks' gestation comes to the physician because of a 3-month history of nausea and vomiting. She has not had any prenatal care. Physical examination shows a uterus consistent in size with a 20-week gestation. Her serum (3-human chorionic gonadotropin concentration is markedly increased. Urinalysis shows 4+ protein. Ultrasonography of the abdomen shows echogenic material within the endometrial cavity but no fetus. The most likely cause of her condition is proliferation of which of the following? O A) Allantois O B) Blastocysts C) Decidua basalis D) Decidua capsularis O E) Trophoblastic tissue

o
Previous

o
Next

m
Lab Values

Review Help

to
Pause

~ m

' V

*"

. V

1:03 PM 28/08/2012

r~'

National Board o f Medical Examiners - Google Chrome https:/?www.starttest.com/api/5.1.1.0/ITDStart.aspx?SVC=f3fd94ad-4a91-4962-b764-2d4caccal061

Exam Section 2: Item 14 o f 50 I Mark

N ational Board o f M edical E xam iners C om prehensive Basic Science Self-Assessment

T im e R em aining: 4 hr 5 m in 34 sec

14. A previously healthy 54-year-old man comes to the clinic because of light-headedness for 6 hours. His symptom began after skiing at a resort at an altitude of 2743 m (9000 ft). He has been taking a carbonic anhydrase inhibitor since 2 days before arrival at the resort. His blood pressure is 110/60 mm Hg while sitting and 95/50 mm Hg while standing. Physical examination shows no other abnormalities. Which of the following is the most likely cause of his orthostatic hypotension? O A) High-altitude sickness O B) Hypovolemia O C) Hypoxia D) Impaired sympathetic nerve activity E) Respiratory alkalosis

r -'

National Board o f Medical Examiners - Google Chrome https://www.starttest.com/api/5.1.1.0/TTDStart.aspx?SVC=f3fd94ad-4a91-4962-b764-2d4caccal061

Exam Section 2: Item 15 o f 50 I Mark

N ational Board o f M edical E xam iners C om prehensive Basic Science Self-Assessment

T im e R em aining: 4 hr 5 m in 28 sec

15. A 55-year-old man comes to the physician because of a 3-day history of fever and shaking chills. He has end-stage renal disease treated with hemodialysis. His temperature is 39.2C (102.6F), pulse is 110/min, and blood pressure is 156/92 mm Hg. Physical examination shows a tunneled central venous catheter entering the right side of the chest below the clavicle; the area surrounding the catheter is not tender. Echocardiography shows a 3-cm vegetation on the tricuspid valve. Which of the following is the most likely causal organism? O A)

Escherichia coli B) Klebsiella pneumoniae C) Staphylococcus aureus D) Streptococcus pyogenes (group A)

O E) Viridans streptococcus

o
Previous

o
Next

m
Lab Values

Review Help

to
Pause -a ^ il

~ m

' V

4
1,5

1:04 PM 28/08/2012

r- ' National Board of Medical Examiners - Google Chrome


Q h ttps://w w w .starttest.eo m /a p i/5 .l.1.0/TTDStart.aspx?SVC=f3fd94ad-4a91-4962-b764-2d4caccal061

Exam Section 2: Item 16 o f 50 H Mark

National Board o f Medical Examiners Comprehensive Basic Science Self-Assessment

Tim e Remaining: 4 hr 5 min 25 sec

16. A 42-year-old woman comes to the physician because of a 3-week history of numbness of the fingers on the left hand. Neurologic examination shows loss of touch graphesthesia and loss of two-point discrimination in the left hand. This patient most likely has a lesion in which of the following locations of the hemispheres shown?

A)

B)

C)

D)

E)

F)

G)

H)

I)

J)

O Previous

o Next

Lab Values

Review

^ Help

to
Pause -a ^ il

~m

' V

i r

4
1,5

1:04 PM 28/08/2012

N ationa: Board o f M ed ica l Examiners - G o o g le C hrom e

[ o

| 0

h ttp s://w w w .sta rtte st.e o m /a p i/5 .l.1.0/TTDStart.aspx?SVC=f3fd94ad-4a91-4962-b764-2d4caccal061

Exam Section 2: Item 17 o f 50 M ark

N ational Board o f M edical E xam iners C om prehensive Basic Science Self-Assessment

T im e R em aining: 4 hr 5 m in 21 sec

17. A 60-year-old woman with a 1-year history of diabetes mellitus has had decreasing vision over the past year. Examination of the eyes shows milky opacity of both lenses. Which of the following aberrant metabolites of glucose is most likely responsible for the formation of this abnormality? 0 A) Fructose O B) Glycogen
0

C) Lactose D) Sorbitol E) Sucrose

o
Previous

o
Next

m
Lab Values

Review Help

to
Pause -a ^ il

~ m

' V

4
1,5

1:04 PM 28/08/2012

r -'

National Board o f Medical Examiners - Google Chrome https://www.starttest.com/api/5.1.1.0/ITDStart.aspx?SVC=f3fd94ad-4a91-4962-b764-2d4caccal061

Exam Section 2: Item 18 o f 50 I Mark

N ational Board o f M edical E xam iners C om prehensive Basic Science Self-Assessment

T im e R em aining: 4 hr 5 m in 19 sec

18. A 22-year-old woman comes to the physician 3 months after she noticed a painless, slowly enlarging mass on the left side of her neck. Physical examination shows a freely mobile, soft, cystic mass with a cutaneous surface opening. The physician explains that it is from incomplete fusion during embryonic development. Which of the following is the most likely location of the opening of the duct leading to the mass in this patient? A) C)
0 0

Anterior to the sternocleidomastoid muscle Postauricular

0 B) Midline on the neck D) Posterior to the parotid gland E) Submental

o
Previous

o
Next

m
Lab Values

Review Help

to
Pause -a ^ il

~ m

' V

4
1,5

1:04 PM 28/08/2012

r -'

National Board o f Medical Examiners - Google Chrome https://www.starttest.com/api/5.1.1.0/TTDStart.aspx?SVC=f3fd94ad-4a91-4962-b764-2d4caccal061

Exam Section 2: Item 19 o f 50 I Mark

N ational Board o f M edical E xam iners C om prehensive Basic Science Self-Assessment

T im e R em aining: 4 hr 5 m in 15 sec

19. In a healthy person, the thyroid gland produces and secretes thyroxine (T4), maintaining metabolism. Which of the following best describes the T4 -binding substance in the blood and the type of cellular receptor for T 4 in the peripheral tissues?

T4-binding Substance in the Blood


A) B) C) D) E) F) Globulin Globulin Globulin Lipid Lipid Lipid

Receptor That Interacts With T4in the Peripheral Tissues


cell membrane receptor mitochondrial nuclear cell membrane receptor mitochondrial nuclear

O
Previous

o
Next

m
Lab Values

Review Help

to
Pause -a ^ il

~ m

' V

4
1,5

1:04 PM 28/08/2012

N a tio n a l Board o f M ed ica l Examiners - G o o g le C hrom e

I0

|i- ^ J

h ttp s://w w w .sta rtte st.e o m /a p i/5 .l.1.0/TTDStart.aspx?SVC=f3fd94ad-4a91-4962-b764-2d4caccal061

Exam Section 2: Item 20 o f 50 Mark

National Board o f Medical Examiners Comprehensive Basic Science Self-Assessment

Tim e Remaining: 4 hr 5 min 3 sec

20. A 42-year-old man has an autoimmune disorder resulting in proximal muscle weakness of the lower extremities. Arrows in the photomicrograph shown indicate membranes that contain high concentrations of channels that are targeted by this condition. Which of the following is the most likely diagnosis? A) Multiple sclerosis B) Myasthenia gravis
o
Previous

o
Next

<P
Lab Values

Review

O
Help

to
Pause -a ^ il

~ m

' V

4
1,5

1:04 PM 28/08/2012

r -'

National Board o f Medical Examiners - Google Chrome https://www.starttest.com/api/5.1.1.0/ITDStart.aspx?SVC=f3fd94ad-4a91-4962-b764-2d4caccal061

I1t= I IB I251

Exam Section 2: Item 20 o f 50 Mark

National Board o f Medical Examiners Comprehensive Basic Science Self-Assessment

Tim e Remaining: 4 hr 4 min 55 sec

20. A 42-year-old man has an autoimmune disorder resulting in proximal muscle weakness of the lower extremities. Arrows in the photomicrograph shown indicate membranes that contain high concentrations of channels that are targeted by this condition. Which of the following is the most likely diagnosis? A) Multiple sclerosis B) Myasthenia gravis C) Myasthenic (Lambert-Eaton) syndrome D) Periodic paralysis E) Tetany

o o ~m H I S s h 'V ir g r r i j ?
Previous Next

Lab Values

Review

Help

Pause

il

^4
^

1,5

1:05 PM 28/08/2012

r -'

National Board o f Medical Examiners - Google Chrome https://www.starttest.com/api/5.1.1.0/ITDStart.aspx?SVC=f3fd94ad-4a91-4962-b764-2d4caccal061

Exam Section 2: Item 21 o f 50 I Mark

N ational Board o f M edical E xam iners C om prehensive Basic Science Self-Assessment

T im e R em aining: 4 hr 4 m in 49 sec

21. A 43-year-old man comes to the emergency department because of a 1-year history of low back pain. Before the examination, the patient says, "My physician is arrogant and insensitive. He never returns my phone calls, I always have to wait forever to be seen, the tests he orders are painful and unnecessary, and he can never tell me what is causing my back pain or how to treat it." Which of the following is the most appropriate response by the emergency department physician about this patient's complaints? A) Reassure the patient that his physician's behavior is not unusual and that low back pain can be difficult to assess B) Encourage the patient to make an appointment with his physician to communicate his concerns C) Explain to the patient how he may register a complaint against his physician with the state medical licensing board D) Provide the patient with a copy of the Patients' Bill of Rights E) Telephone the patient's physician to make him aware that the patient is very dissatisfied with treatment

o
Previous

o
Next

m
Lab Values

Review Help

to
Pause -a ^ il

~ m

' V

4
1,5

1:05 PM 28/08/2012

r -'

National Board o f Medical Examiners - Google Chrome https://www.starttest.com/api/5.1.1.0/ITDStart.aspx?SVC=f3fd94ad-4a91-4962-b764-2d4caccal061

Exam Section 2: Item 22 o f 50 I Mark

N ational Board o f M edical E xam iners C om prehensive Basic Science Self-Assessment

T im e R em aining: 4 hr 4 m in 46 sec

22. A 55-year-old man recovered from a stroke 2 months ago. He lives alone. He is at increased risk for which of the following psychiatric disorders? A) Agoraphobia B) Major depressive disorder C) Obsessive-compulsive disorder D) Post-traumatic stress disorder E) Social phobia

o
Previous

o
Next

m
Lab Values

Review

* *
Help

to
Pause -a ^ il

~ m

' V

4
1,5

1:05 PM 28/08/2012

r -'

National Board o f Medical Examiners - Google Chrome https://www.starttest.com/api/5.1.1.0/ITDStart.aspx?SVC=f3fd94ad-4a91-4962-b764-2d4caccal061

Exam Section 2: Item 23 o f 50 I Mark

N ational Board o f M edical E xam iners C om prehensive Basic Science Self-Assessment

T im e R em aining: 4 hr 4 m in 41 sec

23. A 44-year-old man with a 1-year history of angina pectoris comes to the emergency department because of increasingly severe chest pain during the past 2 days. He has had five previous similar episodes, which required treatment with increasing doses of nitroglycerin to resolve. His temperature is 37C (98.6F), pulse is 105/min, respirations are 16/min, and blood pressure is 150/90 mm Hg. Cardiac examination shows an S 4. An ECG shows ST-segment depression in the precordial leads. In addition to aspirin, heparin, and nitroglycerin, he is given a dose of a monoclonal antibody against the platelet llb/llla receptor. This antibody will most likely prevent binding of which of the following substances to platelets? O A) Adenosine B) ADP C) Fibrinogen D) Serotonin E) Thrombin F) Thromboxane A 2

o
Previous

o
Next

m
Lab Values

Review Help

to
Pause -a ^ il

~ m

' V

4
1,5

1:05 PM 28/08/2012

N ational Board o f Medical Examiners - Google Chrome Q https:/?www.starttest.com/api/5.1.1.0/ITDStart.aspx?SVC=f3fd94ad-4a91-4962-b764-2d4caccal061

Exam Section 2: Item 24 o f 50 I Mark

N ational Board o f M edical E xam iners C om prehensive Basic Science Self-Assessment

T im e R em aining: 4 hr 4 m in 35 sec

24. A 45-year-old woman has a 2-week history of a heart murmur and hepatomegaly. She has a 3-month history of flushing with hypotension and crampy diarrhea. Which of the following is the most likely site of a primary neoplasm in this patient? A) Adrenal gland

B) Bone marrow C) Kidney D) Ovary E) Small intestine

Previous

Lab Values

Review

P1
Help
*

Pause

O
..ill ><>

'RH m "s ;r% IPX f

1:05 PM 28/08/2012

National Beard of Medical Examiners - Google Chrome


Q h ttp s://w w w .sta rtte st.e o m /a p i/5 .l.1.0/TTDStart.aspx?SVC=f3fd94ad-4a91-4962-b764-2d4caccal061

I o

I@

Exam Section 2: Item 25 o f 50 H Mark

National Board o f Medical Examiners Comprehensive Basic Science Self-Assessment

Tim e Remaining: 4 hr 4 min 19 sec

25. A 77-year-old woman dies in the hospital after a long illness. Her vertebral column, obtained at autopsy, is shown in the photograph. The process shown is most likely associated with an increase in which of the following? O A) Calcium O B) Estrogen O C) Interleukin-1 (IL-1) D) Monoclonal immunoglobulin O E) Vitamin D

? T

Previous

Next

Lab Values

(S'

Review

Help

s, - o jd jia i
Pause

r -'

National Board o f Medical Examiners - Google Chrome https://www.starttest.com/api/5.1.1.0/ITDStart.aspx?SVC=f3fd94ad-4a91-4962-b764-2d4caccal061

Exam Section 2: Item 26 o f 50 I Mark

N ational Board o f M edical E xam iners C om prehensive Basic Science Self-Assessment

T im e R em aining: 3 hr 52 m in 22 sec

26. Two clinical trials, Trial X and Trial Y, are conducted to assess the potential therapeutic efficacy of a new experimental antibiotic compared to the currently used antibiotic for treatment of urinary tract infections. The two studies are identical, except the statistical power is 0.8 in Study X, and 0.9 in Study Y. Which of the following is the most accurate conclusion regarding the likelihood of a type II error? O A) Greater in Trial X than in Trial Y O B) Greater in Trial Y than in Trial X C) The same in Trial X and in Trial Y D) Cannot be calculated from the data given

o
Previous

o
Next

m
Lab Values

Review Help

to
Pause -a ^ il

~ m

' V

4
1,5

1:18 PM 28/08/2012

r -'

National Board o f Medical Examiners - Google Chrome https://www.starttest.com/api/5.1.1.0/ITDStart.aspx?SVC=f3fd94ad-4a91-4962-b764-2d4caccal061

Exam Section 2: Item 27 o f 50 I Mark

N ational Board o f M edical E xam iners C om prehensive Basic Science Self-Assessment

T im e R em aining: 3 hr 52 m in 12 sec

27. A 3-year-old boy with a history of unexplained fever, lack of perspiration, absence of response to noxious stimuli, and self-mutilating behavioris diagnosed withcongenital insensitivity to pain with anhidrosis. Genetic analysis shows a missense mutation of the tyrosine kinasedomain of the TrkA gene. Assuming this is theonly signaling defectin this patient, which of the following processes has most likely been disrupted? O A) Binding of nerve growth factor to its receptor B) Formation of TrkA homodimer in response to nerve growth factor C) Phosphorylation of downstream molecules in response to nerve growth factor D) Retrograde transport of nerve growth factor from nerve terminals O E) Synthesis of nerve growth factor

o
Previous

o
Next

m
Lab Values

Review Help

to
Pause -a ^ il

~ m

' V

4
1,5

1:18 PM 28/08/2012

r -'

National Board o f Medical Examiners - Google Chrome https://www.starttest.com/api/5.1.1.0/ITDStart.aspx?SVC=f3fd94ad-4a91-4962-b764-2d4caccal061

Exam Section 2: Item 28 o f 50 I Mark

N ational Board o f M edical E xam iners C om prehensive Basic Science Self-Assessment

T im e R em aining: 3 hr 45 m in 8 sec

28. A 4-year-old boy is brought to the emergency department because of a 1-hour history of headache, nausea and vomiting, and dizziness. Physical examination shows severe lethargy. His venous blood is bright red; carboxyhemoglobin saturation is 30% (reference range < 5%). Which of the following steps in the electron transport pathway is primarily affected in this patient? A Reduced substrate , n -t

Flavin mononucleotide A) B) C) D) E)

o
Previous

o
Next

<P
Lab Values

Review

P
Help

to
Pause i-u. 4 ^ il 1,5 1:25 PM 28/08/2012

r "5

' v

i r

r @

N ationa: Board o f M ed ica l Examiners - G o o g le C hrom e

[ o

| 0

h ttp s://w w w .sta rtte st.e o m /a p i/5 .l.1.0/TTDStart.aspx?SVC=f3fd94ad-4a91-4962-b764-2d4caccal061

Exam Section 2: Item 29 o f 50 M ark

N ational Board o f M edical E xam iners C om prehensive Basic Science Self-Assessment

T im e R em aining: 3 hr 44 m in 25 sec

29. A 44-year-old woman has had pain, swelling, and stiffness in both hands over the past 6 months. The pain and stiffness are worst in the mornings, but symptoms improve with activity. Examination shows painless nodules on the extensor surfaces of the arms and tenderness and swelling of the metacarpophalangeal and proximal interphalangeal joints. Which of the following is involved in the pathologic process affecting her joints? A) Bacterial invasion B) Caseating necrosis C) Crystal deposition D) Necrotizing vasculitis O E) Pannus formation

o
Previous

o
Next

m
Lab Values

Review Help

to
Pause -a ^ il

~ m

' V

4
1,5

1:26 PM 28/08/2012

National Board o f Medical Examiners - Google Chrome Q h ttp s://w w w .sta rtte st.e o m /a p i/5 .l.1.0/TTDStart.aspx?SVC=f3fd94ad-4a91-4962-b764-2d4caccal061

Exam Section 2: Item 30 o f 50 H Mark

National Board o f Medical Examiners Comprehensive Basic Science Self-Assessment

Tim e Remaining: 3 hr 42 min 58 sec

30. A 30-year-old woman and her 25-year-old husband come to the physician for advice about their risk for having another child with Tay-Sachs disease. Their 6-month-old son was recently diagnosed with this disease. There is a history of this disease in both the woman's and the man's families. Which of the following best represents the risk for this couple's next child to have Tay-Sachs disease? A) <1% B) 10% C) 25% D) 50% E) 100%

o
Previous

o
Next

m
Lab Values

Review Help

to
Pause -a ^ il

~ m

' V

4
1,5

1:28 PM 28/08/2012

National Board o f Medical Examiners - Google Chrome Q https:/?www.starttest.com/api/5.1.1.0/TTDStart.aspx?SVC=f3fd94ad-4a91-4962-b764-2d4caccal061

Exam Section 2: Item 31 o f 50 I Mark

National Board o f Medical Examiners Comprehensive Basic Science Self-Assessment

Tim e Remaining: 3 hr 37 min 57 sec

31. A 15-year-old boy is brought to the physician because of painless swelling in his left breast for 1 month. There is no family history of breast cancer. He is at the 65th percentile for height and 70th percentile for weight. His pulse is 65/min, and blood pressure is 118/76 mm Hg. Physical examination shows mild facial acne, thickeningof mustache hairon the face, and the presence of axillary hair. There is no adenopathy. The left breast appears mildly enlarged with a palpable breast bud; the right breast is normal with nogynecomastia. Hisgenitalia are Tanner stage 3. Which of the following is the most appropriate initial statement by the physician regarding this patient's breast finding? A) "Have you tried smoking marijuana recently? That has been known to cause breast enlargement." B) "Some teenagers experiment with their parents' prescription drugs, even birth control pills. Are you?" O C) "This is a common condition for teenage boys and should resolve in time." O D) "This may be a sign of a chromosomal problem. We need to do some genetic testing." O E) "This may be a sign of a more serious condition. We'll have to do more tests to be sure."

National Board o f Medical Examiners - Google Chrome Q https:/?www.starttest.com/api/5.1.1.0/TTDStart.aspx?SVC=f3fd94ad-4a91-4962-b764-2d4caccal061

Exam Section 2: Item 32 o f 50 I Mark

N ational Board o f M edical E xam iners C om prehensive Basic Science Self-Assessment

T im e R em aining: 3 hr 37 m in 29 sec

32. A 20-year-old man comes to the physician because of cramping abdominal pain and diarrhea during the past 3 weeks; he has had a 4.5-kg (10-lb) weight loss during this period. The pain is exacerbated following meals. He went on a camping trip in upstate New York 3 weeks ago, swimming in the nearby lakes and hiking in the mountains. His vital signs are normal. Physical examination shows no abnormalities. Which of the following diagnostic tests is most likely to identify the causal organism of this patient's condition? A) Culture of the stool for enteric bacterial pathogens B) Electron microscopy of the stool for small round viruses C) Microscopic examination of the stool for ova and parasites D) Polymerase chain reaction test of the stool for Shiga toxin E) Proctoscopy and rectal biopsy

o
Previous

o
Next

<P
Lab Values

Review

* *
Help

to
Pause i-u. 4 ^ il 1,5 1:33 PM 28/08/2012

r m

r ~

'

National Board o f Medical Examiners - Google Chrome Q https:/?www.starttest.com/api/5.1.1.0/TTDStart.aspx?SVC=f3fd94ad-4a91-4962-b764-2d4caccal061

1>

Exam Section 2: Item 33 o f 50 I Mark

N ational Board o f M edical E xam iners C om prehensive Basic Science Self-Assessment

T im e R em aining: 3 hr 36 m in 41 sec

33. A 12-year-old African American girl is brought to the emergency department by her mother because of a6-hour history of fever and severe rib pain. She has had numerous episodes in the past. Her paternal aunt and uncle have had similar symptoms since youth.Her temperature is 39C (102.2F). The lungs are clear to auscultation. Abdominal examination shows tenderness to palpation. A peripheral blood smear is shown in the photomicrograph. Which of the following is the most likely underlying cause of this patient's symptoms? A) Abnormal folding of the hemoglobin molecule caused by increased Pco2 B) Abnormal hydrogen bonding in the hemoglobin caused by amino acid substitution on the a chain C) Decreased oxygen affinity caused by an amino acid substitution in the hemoglobin a chain D) Erythrocyte fragility caused by an amino acid substitution in glycoproteins on the cell surface E) Polymerization of hemoglobin with hypoxic conditions due to amino acid substitution in the (3 chain

National Board o f Medical Examiners - Google Chrome Q https:/?www.starttest.com/api/5.1.1.0/TTDStart.aspx?SVC=f3fd94ad-4a91-4962-b764-2d4caccal061

Exam Section 2: Item 34 o f 50 I Mark

N ational Board o f M edical E xam iners C om prehensive Basic Science Self-Assessment

T im e R em aining: 3 hr 36 m in 21 sec

34. A 5-year-old boy is brought to the physician by his mother because of a 3-month history of headache, clumsiness, and fatigue. He had a generalized tonic-clonic seizure 1 month ago. His mother says that he often falls while running. Ophthalmologic examination shows ptosis of the left eye and bilateral external ophthalmoplegia. Physical examination shows weakness of the left lower extremity. Serum studies show an increased lactate concentration. Analysis of a calf muscle biopsy specimen obtained from the left lower extremity shows ragged red fibers. Which of the following metabolic processes is most likely impaired in this patient? A) Fatty acid oxidation

O B) Gluconeogenesis O C) Glycolysis D) Oxidative phosphorylation E) Pentose phosphate pathway

National Board o f Medical Examiners - Google Chrome Q https:/?www.starttest.com/api/5.1.1.0/TTDStart.aspx?SVC=f3fd94ad-4a91-4962-b764-2d4caccal061

Exam Section 2: Item 35 o f 50 I Mark

N ational Board o f M edical E xam iners C om prehensive Basic Science Self-Assessment

T im e R em aining: 3 hr 34 m in 9 sec

35. A 48-year-old woman comes to the emergency department because of a 3-hour history of nausea and acute, sharp, right-sided lower abdominal pain that radiates to her back. She says the pain started 30 minutes into her aerobics class and has increased in severity since then. Physical examination shows an enlarged right ovary. The result of a urine pregnancy test is negative. Ultrasonography of the pelvis confirms the presence of an enlarged ovary, and Doppler ultrasonography shows decreased adnexal flow. Which of the following structures is most likely affected in this patient? O A) Mesometrium O B) Oviduct O C) Round ligament of the uterus D) Suspensory ligament E) Transverse cervical ligament

National Board o f Medical Examiners - Google Chrome Q h ttps://w w w .starttest.eo m /a p i/5 .l.1.0/TTDStart.aspx?SVC=f3fd94ad-4a91-4962-b764-2d4caccal061

Exam Section 2: Item 36 o f 50 I Mark

National Board o f Medical Examiners Comprehensive Basic Science Self-Assessment

Tim e Remaining: 3 hr 29 min 12 sec

36. A 60-year-old man comes to the physician because of skin blistering for 1 week. Six months ago, he had blisters in his mouth that easily ruptured. Physical examination shows scattered blisters over the upper and lower extremities and trunk intermingled with crusted denuded areas. A biopsy specimen of an early lesion shows intraepidermal vesicle formation. Direct immunofluorescence microscopy of intact skin shows deposits of IgG on keratinocytes. Which of the following structures is most likely involved in the skin disorder in this patient? A) Adherens junctions (zonulae adherentes) B) Desmosomes (maculae adherentes) O C) Gap junctions O D) Hemidesmosomes E) Tight junctions (zonulae occludentes)

National Board o f Medical Examiners - Google Chrome Q h ttp s://w w w .sta rtte st.e o m /a p i/5 .l.1.0/ITDStart.aspx?SVC=f3fd94ad-4a91-4962-b764-2d4caccal061

Exam Section 2: Item 37 o f 50 I Mark

National Board o f Medical Examiners Comprehensive Basic Science Self-Assessment

Tim e Remaining: 3 hr 28 min 12 sec

37. A 20-year-old man has cramping periumbilical pain. After several hours, the pain shifts to the right lower quadrant and becomes constant. His temperature is 40C (104F), and his leukocyte count is 20,000/mm3with 90% neutrophils. Which of the following mediators is most likely causing this patient's fever and leukocytosis? O A) Interleukin-1 (IL-1) O B) lnterleukin-2 (IL-2) C) Leukotriene B4 D) Leukotriene C 4 E) Prostaglandin D2

Previous

Next

Lab Values

< P

Review

&
Help

to
Pause ^ *

rm

E iiw

^ jr z j

*"

. V

1:43 PM 28/08/2012

N ational Board o f Medical Examiners - Google Chrome Q h ttp s://w w w .sta rtte st.e o m /a p i/5 .l.1.0/TTDStart.aspx?SVC=f3fd94ad-4a91-4962-b764-2d4caccal061

Exam Section 2: Item 38 o f 50 H Mark

National Board o f Medical Examiners Comprehensive Basic Science Self-Assessment

Tim e Remaining: 3 hr 27 min 4 sec

38. A screening test is used to detect prostate cancer in a population of 100,000 men. The test has a sensitivity of 70% and a specificity of 90%. The test is then used to screen a second population in which the prevalence of prostate cancer is different. Which of the following is the most likely effect on the positive predictive value of this test? A) Decreases as prevalence increases B) Increases as prevalence decreases C) Increases as prevalence increases D) No change as prevalence changes

Previous

p 1
Lab Values

Review

Pause

C l
1:44 PM 28/08/2012

National Board o f Medical Examiners - Google Chrome Q https:/?www.starttest.com/api/5.1.1.0/TTDStart.aspx?SVC=f3fd94ad-4a91-4962-b764-2d4caccal061

Exam Section 2: Item 39 o f 50 I Mark

N ational Board o f M edical E xam iners C om prehensive Basic Science Self-Assessment

T im e R em aining: 3 hr 23 m in 50 sec

39. An investigator is studying the human immune response to tumor cell antigens in malignancies. Which of the following sets of cancer types and tumor antigens is most likely to produce the highest antibody titer?

Cancer Type
A) B) C) D) E) B-cell lymphoma Breast cancer Cervical cancer Melanoma Prostate cancer

Tumor Antigen
CD19 HER2/neu human papillomavirus type 16 E6 protein tyrosinase prostatic acid phosphatase

National Board o f Medical Examiners - Google Chrome Q https:/?www.starttest.com/api/5.1.1.0/TTDStart.aspx?SVC=f3fd94ad-4a91-4962-b764-2d4caccal061

Exam Section 2: Item 40 o f 50 I Mark

N ational Board o f M edical E xam iners C om prehensive Basic Science Self-Assessment

T im e R em aining: 3 hr 23 m in 22 sec

40. A previously healthy 35-year-old man comes to the physician because of a 2-year history of gradually decreasing libido and intermittent inability to achieve erection or orgasm; he also has had decreased growth of facial hair during this period. Puberty began at the age of 12 years. He takes no medications. He does not smoke or drink alcohol. He is 180 cm (5 ft 11 in) tall and weighs 75 kg (165 lb); BMI is 23 kg/m2. His vital signs are within normal limits. Physical examination shows a typical male-pattern distribution of hair. Genital examination shows no abnormalities. Serum studies show a thyroid-stimulating hormone concentration of 2 pU/mL, prolactin concentration of 120 ng/ml_, and total testosterone concentration of 25 nmol/L (N=10-35). Semen analysis shows a markedly decreased sperm count with impaired motility. An MRI of the brain shows a 1-cm lesion in the pituitary gland. Administration of which of the following is the most appropriate treatment for this patient? O A) Bromocriptine B) Gonadotropin-releasing hormone O C) Human chorionic gonadotropin O D) Sildenafil O E) Testosterone O F) Thyroxine

National Board o f Medical Examiners - Google Chrome Q h ttp s://w w w .sta rtte st.e o m /a p i/5 .l.1.0/TTDStart.aspx?SVC=f3fd94ad-4a91-4962-b764-2d4caccal061

Exam Section 2: Item 41 o f 50 H Mark

National Board o f Medical Examiners Comprehensive Basic Science Self-Assessment

Tim e Remaining: 3 hr 21 min 0 sec

41. A 3-year-old girl is found to have a grade 4/6, loud, harsh, high-pitched holosystolic murmur that radiates over the precordium and a palpable thrill at the left sternal border. Which of the following defects is most likely in this patient? A) Aortic regurgitation B) Aortic stenosis C) Atrial septal defect D) Coarctation of the aorta E) Mitral regurgitation F) Mitral stenosis G) Patent ductus arteriosus H) Pulmonic stenosis I) Tricuspid regurgitation J) Ventricular septal defect

Previous

p 1
Lab Values

Review

Pause

C l
1:50 PM 28/08/2012

National Board o f Medical Examiners - Google Chrome Q https:/?www.starttest.com/api/5.1.1.0/TTDStart.aspx?SVC=f3fd94ad-4a91-4962-b764-2d4caccal061

Exam Section 2: Item 42 o f 50 I Mark

N ational Board o f M edical E xam iners C om prehensive Basic Science Self-Assessment

T im e R em aining: 3 hr 17 m in 28 sec

42.

A 43-year-old woman with metastatic colon cancer has maintained remission by taking bevacizumab as a single agent. The ability to administer this agent despite it being a foreign protein is most likely a result of which of the following? A) The agent being a humanized antibody B) The agent containing the variable region of a murine antibody molecule directed against vascular endothelial growth factor

O C) B-cell dysfunction D) The immunosuppressive effect of previously administered chemotherapy E)T-cell dysfunction

National Board o f Medical Examiners - Google Chrome Q h ttp s://w w w .sta rtte st.e o m /a p i/5 .l.1.0/TTDStart.aspx?SVC=f3fd94ad-4a91-4962-b764-2d4caccal061

Exam Section 2: Item 43 o f 50 H Mark

National Board o f Medical Examiners Comprehensive Basic Science Self-Assessment

Tim e Remaining: 3 hr 17 min 7 sec

43. A 23-year-old man comes to the physician because of a low-grade fever, nasal congestion, and muscle aches for 1 week. He states that his girlfriend recently told him that his eyes appeared yellow. He has not had clay-colored stools or dark urine. His only medication is ibuprofen. He has no recent history of foreign travel, tattoos, use of intravenous drugs, or exposure to blood products. He is 183 cm (6 ft) tall and weighs 77 kg (170 lb); BMI is 23 kg/m2. Physical examination shows mild scleral icterus but no subungual icterus. The remainder of the examination shows no abnormalities. Serum studies show: Bilirubin, total Direct Alkaline phosphatase AST ALT Which of the following is the most likely diagnosis? A) Gilbert syndrome B) Hepatitis D C) Hereditary spherocytosis D) Idiopathic cirrhosis E) Steatohepatitis 2.6 mg/dL 0.3 mg/dL 30 U/L 12 U/L 10 U/L

National Board o f Medical Examiners - Google Chrome Q h ttp s://w w w .sta rtte st.e o m /a p i/5 .l.1.0/ITDStart.aspx?SVC=f3fd94ad-4a91-4962-b764-2d4caccal061

Exam Section 2: Item 44 o f 50 H Mark

National Board o f Medical Examiners Comprehensive Basic Science Self-Assessment

Tim e Remaining: 3 hr 16 min 45 sec

44. A 32-year-old woman comes to the physician because of a 1-month history of progressive shortness of breath and anxiety. She says that her symptoms become much more pronounced as the day progresses. She is 170 cm (5 ft 7 in) tall and weighs 70 kg (155 lb); BMI is 24 kg/m2. Breath sounds are normal on auscultation of the chest, but the patient has difficulty taking a long, deep breath. Arterial blood gas analysis on room air shows: pH Pco2 Po2 7.33 70 mm Hg 65 mm Hg

Pulmonary function testing shows a decreased vital capacity, tidal volume, and expiratory reserve volume. Her residual volume is within the reference range. Which of the following is the most likely underlying cause of this patient's condition? O A) a ^Antitrypsin deficiency B) Central nervous system neoplasm O C) Chronic bronchitis O D) Chronic opiate use E) Myasthenia gravis F) Obesity

National Board o f Medical Examiners - Google Chrome Q https:/?www.starttest.com/api/5.1.1.0/TTDStart.aspx?SVC=f3fd94ad-4a91-4962-b764-2d4caccal061

Exam Section 2: Item 45 o f 50 I Mark

N ational Board o f M edical E xam iners C om prehensive Basic Science Self-Assessment

T im e R em aining: 3 hr 14 m in 45 sec

45. An 18-year-old man has had temperatures to 38.3C (101F), a sore throat, and cervical lymph node enlargement for 8 days. A latex agglutination test result for Epstein-Barr virus antibody is positive. Atypical cells with abundant lacey cytoplasm in the peripheral blood smear are most likely derived from which of the following types of cells? O A) B lymphocytes O B) Basophils O C) Neutrophils O D) Plasma cells O E) T lymphocytes

National Board o f Medical Examiners - Google Chrome Q https:/?www.starttest.com/api/5.1.1.0/TTDStart.aspx?SVC=f3fd94ad-4a91-4962-b764-2d4caccal061

Exam Section 2: Item 46 o f 50 I Mark

N ational Board o f M edical E xam iners C om prehensive Basic Science Self-Assessment

T im e R em aining: 3 hr 13 m in 11 sec

46. An 18-month-old boy is brought to the physician because of progressive facial abnormalities and slow growth during the past 6 months. He has an older brother with similar abnormalities who also has mental retardation. The patient is below the 3rd percentile for length and at the 10th percentile for weight. Physical examination shows coarse facial features, hepatosplenomegaly, and joint stiffness. His urine concentrations of dermatan sulfate and heparan sulfate are increased. A deficiency of which of the following enzyme activities is the most likely cause of the findings in this patient? A) Galactosidase

O B) Glycosylase C) Hexosaminidase A D) Iduronate sulfatase O E) Lipoxygenase

National Board o f Medical Examiners - Google Chrome Q https:/?www.starttest.com/api/5.1.1.0/TTDStart.aspx?SVC=f3fd94ad-4a91-4962-b764-2d4caccal061

Exam Section 2: Item 47 o f 50 I Mark

N ational Board o f M edical E xam iners C om prehensive Basic Science Self-Assessment

T im e R em aining: 3 hr 10 m in 24 sec

47. A 6-year-old boy is brought to the physician by his mother because of a 2-day history of fever, sore throat, and occasional vomiting. His temperature is 39C (102.2F). Physical examination shows erythema and purulent exudate over the posterior pharyngeal wall and tonsils. The anterior cervical glands are enlarged and tender. A throat culture grows group A Streptococcus. Because he is allergic to penicillin, treatment with erythromycin is begun. His condition improves during the next 2 days. The effectiveness of this antibiotic is most likely due to which of the following actions in protein synthesis? A) Causes misreading of mRNA information B) Functions as an analog of aminoacyl-tRNA C) Inactivates elongation factor-2 D) Interferes in aminoacyl-tRNA binding E) Interferes with translocation

National Board o f Medical Examiners - Google Chrome Q https:/?www.starttest.com/api/5.1.1.0/TTDStart.aspx?SVC=f3fd94ad-4a91-4962-b764-2d4caccal061

Exam Section 2: Item 48 o f 50 I Mark

N ational Board o f M edical E xam iners C om prehensive Basic Science Self-Assessment

T im e R em aining: 3 hr 9 m in 26 sec

48.

A male newborn at 34 weeks' gestation is delivered vaginally. Physical examination shows no abnormalities except for a 1-cm, raised, red area on the anterior section of the neck. His mother is concerned, and the physician explains that the lesion is a hemangioma, a benign growth. This neoplasm most likely originated in which of the following tissues?

O A) Ectoderm B) D) Endothelium Notochord O C) Neural crest cells E) Yolk sac

National Board o f Medical Examiners - Google Chrome Q https:/?www.starttest.com/api/5.1.1.0/TTDStart.aspx?SVC=f3fd94ad-4a91-4962-b764-2d4caccal061

Exam Section 2: Item 49 o f 50 I Mark

N ational Board o f M edical E xam iners C om prehensive Basic Science Self-Assessment

T im e R em aining: 3 hr 8 m in 11 sec

49. A researcher in a pharmaceutical company designs a new protease inhibitor that inhibits replication of HIV in T lymphocytes in culture. In subsequent assays, which of the following findings is most likely to indicate that the compound is workingspecifically as a protease inhibitor? A) The drug prevents integration of proviral DNA into the host genome B) The RNA is partially reverse transcribed into proviral DNA C) There is a lack of a mature core D) Transcription from the HIV promoter is blocked E) The virus does not bind to CD4 in the presence of the drug

o
Previous

o
Next

<P
Lab Values

#
Review

O
Help

to
Pause -a ^ il

4
^

r m

r ~

'

2:03 PM 28/08/2012

National Board o f Medical Examiners - Google Chrome Q https:/?www.starttest.com/api/5.1.1.0/TTDStart.aspx?SVC=f3fd94ad-4a91-4962-b764-2d4caccal061

Exam Section 2: Item 50 o f 50 I Mark

N ational Board o f M edical E xam iners C om prehensive Basic Science Self-Assessment

T im e R em aining: 3 hr 7 m in 30 sec

50.

A 42-year-old man seeks medical attention because of the gradual onset of shortness of breath, night sweats, weight loss, and fatigue. An x-ray of the chest shows enlargement of the hilar lymph nodes and nodular infiltrates in both lungs. A PPD skin test is negative. Examination of lung tissue obtained on biopsy shows multiple noncaseating granulomas with many giant cells. No organisms are seen on sections stained with hematoxylin and eosin, acid-fast stain, or silver stain. Which of the following is the most likely diagnosis?

O A)Asbestosis B) Idiopathic pulmonary fibrosis C)

Pneumocystis jiroveci (formerly P. carinii) pneumonia

O D) Sarcoidosis O E) Tuberculosis

National Board o f Medical Examiners - Google Chrome Q h ttp s://w w w .sta rtte st.e o m /a p i/5 .l.1.0/TTDStart.aspx?SVC=f3fd94ad-4a91-4962-b764-2d4caccal061

Exam Section 3: Item 1 o f 50 H Mark

National Board o f Medical Examiners Comprehensive Basic Science Self-Assessment

Tim e Remaining: 4 hr 19 min 29 sec

1. A 65-year-old man dies 4 days after complete occlusion of the left coronary artery. The left ventricle contains a large irregular area that is soft and pale. Which of the following processes is most likely responsible? A) Caseation necrosis B) Coagulation necrosis C) Dystrophic calcification D) Fat necrosis E) Gangrenous necrosis O F) Liquefaction necrosis G) Metastatic calcification

p 1
Lab Values

Review

Pause
2:18 PM 28/08/2012

Cl

National Board o f Medical Examiners - Google Chrome Q https:/?www.starttest.com/api/5.1.1.0/TTDStart.aspx?SVC=f3fd94ad-4a91-4962-b764-2d4caccal061

Exam Section 3: Item 2 o f 50 I Mark

N ational Board o f M edical E xam iners C om prehensive Basic Science Self-Assessment

T im e R em aining: 4 hr 18 m in 38 sec

2. A 23-year-old woman comes to the physician because of a 3-week history of persistent cough that is worsened by the cold air in her drafty apartment. She does not smoke, takes no medications, and has no history of asthma or chronic pulmonary disease. Physical examination shows no abnormalities. Short-term cough suppressant treatment is initiated with a drug that does not cause constipation and has a low potential for substance abuse. This drug is most likely which of the following? A) Codeine B) Dextromethorphan C) Hydromorphone D) Oxycodone E) Tramadol

National Board o f Medical Examiners - Google Chrome Q https:/?www.starttest.com/api/5.1.1.0/TTDStart.aspx?SVC=f3fd94ad-4a91-4962-b764-2d4caccal061

Exam Section 3: Item 3 o f 50 I Mark

N ational Board o f M edical E xam iners C om prehensive Basic Science Self-Assessment

T im e R em aining: 4 hr 17 m in 35 sec

3. A 26-year-old woman comes to the physician's office because of a 13.5-kg (30-lb) weight gain, irregular menstrual periods, constipation, and fatigue for the past 6 months. She also complains of increased difficulties with concentration and memory for the past month. Physical examination shows a blood pressure of 145/84 mm Hg, dry skin, and decreased muscle stretch reflexes at the ankles. Serum cholesterol concentration is 310 mg/dL, and serum potassium concentration is 4.0 mEq/L. Determination of which of the following serum concentrations is most likely to confirm the diagnosis? A) Cortisol B) Dehydroepiandrosterone C) Hemoglobin A 1c D) Luteinizing hormone E) Prolactin F) Testosterone G) Thyroid-stimulating hormone

National Board o f Medical Examiners - Google Chrome Q https:/?www.starttest.com/api/5.1.1.0/TTDStart.aspx?SVC=f3fd94ad-4a91-4962-b764-2d4caccal061

Exam Section 3: Item 4 o f 50 I Mark

N ational Board o f M edical E xam iners C om prehensive Basic Science Self-Assessment

T im e R em aining: 4 hr 16 m in 1 sec

4. A 38-year-old woman is brought to the emergency department after being found lying on the sidewalk. She is conscious but says she is too weak to stand. Physical examination shows a red, swollen, tender tongue; angular stomatitis; and spoon-shaped nails. Hemoglobin and mean corpuscular volume are decreased. Which of the following is the most likely diagnosis? A) Vitamin B 12 (cobalamin) deficiency

B) Folic acid deficiency O C) Iron deficiency D) Sickle cell disease E) Vitamin B 1 (thiamine) deficiency

National Board o f Medical Examiners - Google Chrome Q https:/?www.starttest.com/api/5.1.1.0/TTDStart.aspx?SVC=f3fd94ad-4a91-4962-b764-2d4caccal061

Exam Section 3: Item 5 o f 50 I Mark

N ational Board o f M edical E xam iners C om prehensive Basic Science Self-Assessment

T im e R em aining: 4 hr 15 m in 21 sec

5. A 92-year-old man is brought to the emergency department because of a 6-hour history of chest pain and shortness of breath. A diagnosis of congestive heart failure is made. Diuretic therapy is initiated, and his symptoms resolve over the next 3 days. However, his pulse remains at 44/min, and he has fatigue with minimal exertion. His physician believes that he should receive a pacemaker; however, the patient has expressed a desire to forego all invasive procedures. The physician presents the benefits of the pacemaker fully, but he describes the risks and follow-up in a cursory manner. The patient consents to the procedure. This physician has violated which of the following ethical principles? O A) Beneficence O B) Informed consent C) Justice D) Nonmaleficence E) Veracity

National Board o f Medical Examiners - Google Chrome

Exam Section 3: Item 6 o f 50 I Mark

N ational Board o f M edical E xam iners C om prehensive Basic Science Self-Assessment

T im e R em aining: 4 hr 10 m in 44 sec

6. A 46-year-old man receives the diagnosis of squamous cell carcinoma of the esophagus. A barium swallow is shown. Esophagectomy at the region indicated by the arrows is most likely to involve ligation of arterial branches of which of the following vessels? O A) Aorta B) Internal thoracic arteries C) Pulmonary arteries O D) Superior mesenteric artery E) Thyrocervical trunk

National Board o f Medical Examiners - Google Chrome Q https:/?www.starttest.com/api/5.1.1.0/TTDStart.aspx?SVC=f3fd94ad-4a91-4962-b764-2d4caccal061

Exam Section 3: Item 7 o f 50 I Mark

N ational Board o f M edical E xam iners C om prehensive Basic Science Self-Assessment

T im e R em aining: 4 hr 9 m in 36 sec

7. A 63-year-old woman comes to the physician because of a 2-month history of a drooping left eyelid, intermittent pain of her left eye, and an unusually large pupil. Her husband has told her that the eye appears to be "looking sideways." Ophthalmologic examination shows ptosis, lateral strabismus, and mydriasis of the left eye. An MRI of the brain shows an aneurysm of the left posterior communicating artery. The function of which of the following muscles is most likely to remain intact in this patient? O A) Ciliary O B) Inferior oblique O C) Inferior rectus O D) Pupillary constrictor O E) Superior oblique F) Superior rectus

National Board o f Medical Examiners - Google Chrome Q https:/?www.starttest.com/api/5.1.1.0/TTDStart.aspx?SVC=f3fd94ad-4a91-4962-b764-2d4caccal061

Exam Section 3: Item 8 o f 50 I Mark

N ational Board o f M edical E xam iners C om prehensive Basic Science Self-Assessment

T im e R em aining: 4 hr 8 m in 14 sec

8. A 30-year-old woman is brought to the emergency department 6 hours after the onset of severe abdominal pain and confusion. Her brother, father, and paternal grandmother had similar problems. Two weeks ago, she began a low-calorie diet. Her pulse is 120/min, and blood pressure is 160/110 mm Hg. Physical examination shows restlessness, excess sweating, and tremors. There is no abdominal tenderness. Urine studies show an increased porphobilinogen concentration. Which of the following is the most likely pattern of inheritance causing this patient's condition? O A) Autosomal dominant O B) Autosomal recessive O C) Mitochondrial DNA mutation D) Mutifactorial O E)X-linked dominant F)X-linked recessive

National Board o f Medical Examiners - Google Chrome Q https:/?www.starttest.com/api/5.1.1.0/TTDStart.aspx?SVC=f3fd94ad-4a91-4962-b764-2d4caccal061

Exam Section 3: Item 9 o f 50 I Mark

N ational Board o f M edical E xam iners C om prehensive Basic Science Self-Assessment

T im e R em aining: 4 hr 4 m in 32 sec

9. A 15-year-old boy is brought to the physician by his mother because of a 2-month history of sneezing, coughing, and watery eyes immediately after he mows the lawn. Fifteen minutes after his symptoms first begin, he develops a red rash containing areas that are soft to the touch. Physical examination shows boggy, soft, raised, pruritic, erythematous lesions with wheal and flare eruptions over the face, trunk, and back. Which of the following describes the most likely underlying mechanism of the rash in this patient? A) Type I (immediate) hypersensitivity B) Type II (complement-mediated cytotoxic) hypersensitivity C) Type III (immune complex-mediated) hypersensitivity D) Type IV (delayed) hypersensitivity

National Board o f Medical Examiners - Google Chrome Q https:/?www.starttest.com/api/5.1.1.0/TTDStart.aspx?SVC=f3fd94ad-4a91-4962-b764-2d4caccal061

Exam Section 3: Item 10 o f 50 I Mark

N ational Board o f M edical E xam iners C om prehensive Basic Science Self-Assessment

T im e R em aining: 4 hr 3 m in 32 sec

10. A 95-year-old man who is a resident of a skilled nursing care facility is brought to the emergency department because of a 1-day history of temperatures to 39.4C (102.9F), headache, cough, and muscle aches. Several other residents have similar symptoms. Physical examination shows no other abnormalities. Treatment with a neuraminidase inhibitor is begun. This drug most likely will inhibit which of the following processes, therefore decreasing the duration of this patient's symptoms? A) Nucleocapsid-matrix protein interactions B) Release of virus from infected epithelial cells O C) Replication of genomic RNA D) Synthesis of surface glycoproteins O E) Transcription of mRNA

National Board o f Medical Examiners - Google Chrome Q https:/?www.starttest.com/api/5.1.1.0/TTDStart.aspx?SVC=f3fd94ad-4a91-4962-b764-2d4caccal061

Exam Section 3: Item 11 o f 50 I Mark

N ational Board o f M edical E xam iners C om prehensive Basic Science Self-Assessment

T im e R em aining: 4 hr 2 m in 51 sec

11. A 32-year-old man comes to the physician because of back pain and joint stiffness for 3 months. Hissymptoms improve aftermild exercise.During thisillness,he alsohas had fatigue and a 3.2-kg (7-lb) weight loss. Physical examination and x-rays of the sacroiliac joints confirm a diagnosis of ankylosing spondylitis. Which of the followinglaboratory tests would be most appropriate for the patient's two children to evaluate their susceptibility to this disease? A) Analysis of class I MHC allele expression B) Measurement of B-lymphocyte mitogen responses to pokeweed mitogen C) Measurement of T-lymphocyte mitogen responses to phytohemagglutinin D) Polymerase chain reaction assay for

bcl-2 gene expression

E) Polymerase chain reaction assay for tyrosine kinase gene expression

National Board o f Medical Examiners - Google Chrome Q https:/?www.starttest.com/api/5.1.1.0/TTDStart.aspx?SVC=f3fd94ad-4a91-4962-b764-2d4caccal061

Exam Section 3: Item 12 o f 50 0 M ark

N ational Board o f M edical E xam iners C om prehensive Basic Science Self-Assessment

T im e R em aining: 4 hr 1 m in 20 sec

12.

A 47-year-old woman comes to the physician because she wants to "do something about my wrinkles." Physical examination shows fine wrinkles in photodamaged skin on the face. Treatment with tretinoin is started. Collagen synthesis will most likely be increased in this patient by which of the following mechanisms? A) C) Activation of nuclear gene transcription Displaced vitamin A from cellular stores

O B) Decreased cAMP production O D) Increased sebum production E) Protection of keratinocytes from UVB irradiation

National Board o f Medical Examiners - Google Chrome Q https:/?www.starttest.com/api/5.1.1.0/TTDStart.aspx?SVC=f3fd94ad-4a91-4962-b764-2d4caccal061

Exam Section 3: Item 13 o f 50 I Mark

N ational Board o f M edical E xam iners C om prehensive Basic Science Self-Assessment

T im e R em aining: 3 hr 59 m in 18 sec

13. A 25-year-old man receives a bupivacaine injection for median nerve block as treatment of carpal tunnel syndrome. Which of the following is the mechanism of action of this drug on the membrane? O A) Activation of the Na+-K+ATPase B) Decreased permeability to C a2+ C) Decreased permeability to Na+ D) Increased reuptake glutamate E) Inhibition of the Na+ a2+antiporter /C

National Beard of Medical Examiners - Google Chrome


Q h ttp s://w w w .sta rtte st.e o m /a p i/5 .l.1.0/TTDStart.aspx?SVC=f3fd94ad-4a91-4962-b764-2d4caccal061

|0

Exam Section 3: Item 14 o f 50 H Mark

National Board o f Medical Examiners Comprehensive Basic Science Self-Assessment

Tim e Remaining: 3 hr 58 min 6 sec

14. An experimental model of treatment for sickle cell disease involves reactivating the genes that code for the (3 chains of fetal hemoglobin. This treatment is most likely to increase the affinity of hemoglobin for which of the following? A) 2,3-Bisphosphoglycerate

O B) Carbon dioxide 0 C) Chloride ions D)


0

Oxygen

E) Protons

Previous

< *

Lab Values

Review

Help

Pause

O
* ..ill ><>

2:40 PM 28/08/2012

National Board o f Medical Examiners - Google Chrome Q h ttp s://w w w .sta rtte st.e o m /a p i/5 .l.1.0/TTDStart.aspx?SVC=f3fd94ad-4a91-4962-b764-2d4caccal061

Exam Section 3: Item 15 o f 50 H Mark

National Board o f Medical Examiners Comprehensive Basic Science Self-Assessment

Tim e Remaining: 3 hr 56 min 21 sec

15. A 25-year-old woman with obstructive sleep apnea comes to the physician because of oily, fatty bowel movements since starting a drug 1 month ago to aid in weight loss. She has had a 5-kg (11 -lb) weight loss during this period. She is 157 cm (5 ft 2 in) tall and now weighs 66 kg (145 lb); BMI is 27 kg/m2. Her blood pressure is 130/80 mm Hg; it was 140/85 mm Hg at her last visit 1 month ago. Physical examination shows no other abnormalities. The results of serum lipid studies done 1 month ago and at this visit are shown.

Visit 1 Month Ago


Cholesterol, total (mg/dL) HDL-cholesterol (mg/dL) LDL-cholesterol (mg/dL) Triglycerides (mg/dL) 285 25 250 300

This Visit
225 28 150 200

Which of the following drugs is the most likely cause of the steatorrhea and changes in lipid serum studies in this patient? A) C) Atorvastatin Orlistat

O B) Bupropion O D) Phentermine O E) Sibutramine

r 0

National Board o f Medical Examiners - Google Chrome https://www.starttest.com/api/5.1.1.0/TTDStart.aspx?SVC=f3fd94ad-4a91-4962-b764-2d4caccal061

Exam Section 3: Item 16 o f 50 I Mark

N ational Board o f M edical E xam iners C om prehensive Basic Science Self-Assessment

T im e R em aining: 3 hr 52 m in 25 sec

16. An epidemiologist is investigating an outbreak of diarrhea among a total of 1000 consumers of vegetables. Of those consumers, 800 people ate tomatoes and 200 people ate lettuce. A total of 400 people became ill; 80 of these people ate tomatoes, and 40 people ate lettuce. Which of the following indicates the probability of diarrhea having developed in people who consumed lettuce? A) 0.08 B) 0.1 C) 0.2 D) 0.4 E) 0.5

r 0

National Board o f Medical Examiners - Google Chrome https://www.starttest.com/api/5.1.1.0/TTDStart.aspx?SVC=f3fd94ad-4a91-4962-b764-2d4caccal061

Exam Section 3: Item 17 o f 50 I Mark

N ational Board o f M edical E xam iners C om prehensive Basic Science Self-Assessment

T im e R em aining: 3 hr 46 m in 14 sec

17. A 25-year-old medical student whose father had alcoholism and died of cirrhosis initially gets angry at patients with alcoholism. Over the course of training, he continues to recognize that working with these types of patients is distressing to him, but he learns methods of dealing more effectively with them. Which of the following mental mechanisms best describes his ability to effectively control his emotions? 0 A) Denial O B) Projection O C) Reaction formation O D) Repression O E) Suppression

r 0

National Board o f Medical Examiners - Google Chrome https://www.starttest.com/api/5.1.1.0/TTDStart.aspx?SVC=f3fd94ad-4a91-4962-b764-2d4caccal061

Exam Section 3: Item 18 o f 50 I Mark

N ational Board o f M edical E xam iners C om prehensive Basic Science Self-Assessment

T im e R em aining: 3 hr 45 m in 38 sec

18. A 25-year-old man comes to the physician because of a 4-day history of pain and burning with urination and frequent urinary urgency. He is sexually active and has several female partners; he does not use condoms. His temperature is 37C (98.6F). Urinalysis shows 2+ protein; cultures grow on chocolate agar. A Gram stain shows gram-negative diplococci. The organism causing this patient's symptoms most likely produces which of the following virulence factors? O A) Elementary bodies O B) Exotoxins O C) Free radicals O D) Pili 0 E) Spores

National Board o f Medical Examiners - Google Chrome Q h ttp s://w w w .sta rtte st.e o m /a p i/5 .l.1.0/TTDStart.aspx?SVC=f3fd94ad-4a91-4962-b764-2d4caccal061

1 cz, I IB 1 ^ 3 - 1

Exam Section 3: Item 19 o f 50 M ark

N ational Board o f M edical E xam iners C om prehensive Basic Science Self-Assessment

T im e R em aining: 3 hr 44 m in 11 sec

19. A 60-year-old woman comes to the physician because of a 2-day history of flank pain and pain with urination. She has hypertension, recurrent urinary tract infections, and hypothyroidism. Her temperature is 37C (98.6F), pulse is 96/min, and blood pressure is 152/92 mm Hg. Physical examination shows no other abnormalities. Urinalysis shows 3+ blood, 1+ leukocytes, and few bacteria. Ultrasonography of the abdomen shows a large calculus filling the entire right renal pelvis. The renal calculus in this patient is most likely composed of which of the following substances? A) Calcium oxalate

O B) Cystine O C) Struvite O D) Uric acid

r 0

National Board o f Medical Examiners - Google Chrome https://www.starttest.com/api/5.1.1.0/TTDStart.aspx?SVC=f3fd94ad-4a91-4962-b764-2d4caccal061

Exam Section 3: Item 20 o f 50 I Mark

N ational Board o f M edical E xam iners C om prehensive Basic Science Self-Assessment

T im e R em aining: 3 hr 43 m in 14 sec

20. A female newborn is delivered at 34 weeks' gestation in an advanced-care setting where special delivery systems are available. The diagnosis of persistent pulmonary hypertension is made. Considering that the newborn can be carefully monitored for methemoglobinemia, which of the following is the most appropriate therapy? O A) Desflurane O B) Hyperbaric oxygen O C) Nitric oxide 0 D) Nitrous oxide E) Oxygen diluted with helium

National Beard of Medical Examiners - Google Chrome

[a

[p

Q h ttp s://w w w .sta rtte st.e o m /a p i/5 .l.1.0/TTDStart.aspx?SVC=f3fd94ad-4a91-4962-b764-2d4caccal061

Exam Section 3: Item 21 o f 50 H Mark

National Board o f Medical Examiners Comprehensive Basic Science Self-Assessment

Tim e Remaining: 3 hr 37 min 42 sec

21. An autopsy is done on a 50-year-old man who died of pneumonia despite 5 days of antibiotic therapy in the intensive care unit. He had a 15-year history of alcoholism. A photograph of a sagittal section of the brain is shown. Based on this pathology, which of the following findings was most likely present on neurologic examination of the patient prior to his death? A) Dysdiadochokinesia B) Dysmetria on finger-nose testing C) Essential tremor D) Gait ataxia O E) Present Romberg sign

r 0

National Board o f Medical Examiners - Google Chrome https://www.starttest.com/api/5.1.1.0/ITDStart.aspx?SVC=f3fd94ad-4a91-4962-b764-2d4caccal061

Exam Section 3: Item 22 o f 50 I Mark

N ational Board o f M edical E xam iners C om prehensive Basic Science Self-Assessment

T im e R em aining: 3 hr 36 m in 26 sec

22.

A healthy 56-year-old man comes to the physician for a pre-employment physical. Cardiac examination shows a systolic heart murmur that is best heard in the second interspace at the right sternal border and is transmitted to the carotid arteries. The murmur begins immediately after S 1 rises in crescendo before falling in pitch, and ceases before S 2. Which of the f following is the most likely cause of the murmur?

O A) Aortic stenosis B) Atrial septal defect C) Mitral regurgitation O D) Pulmonary regurgitation O E) Tricuspid stenosis

r 0

National Board o f Medical Examiners - Google Chrome https://www.starttest.com/api/5.1.1.0/TTDStart.aspx?SVC=f3fd94ad-4a91-4962-b764-2d4caccal061

Exam Section 3: Item 23 o f 50 I Mark

N ational Board o f M edical E xam iners C om prehensive Basic Science Self-Assessment

T im e R em aining: 3 hr 35 m in 21 sec

23. A 59-year-old man who has smoked 2 packs of cigarettes daily for 40 years develops an ulcerated laryngeal neoplasm. Examination of tissue obtained on biopsy of the lesion is most likely to show which of the following? O A) Adenocarcinoma 0 B) Reactive nodule C) Squamous cell carcinoma D) Squamous papilloma E) Undifferentiated carcinoma

National Board o f Medical Examiners - Google Chrome Q h ttp s://w w w .sta rtte st.e o m /a p i/5 .l.1.0/TTDStart.aspx?SVC=f3fd94ad-4a91-4962-b764-2d4caccal061

1 cz, I IB 1 ^ 3 - 1

Exam Section 3: Item 24 o f 50 M ark

N ational Board o f M edical E xam iners C om prehensive Basic Science Self-Assessment

T im e R em aining: 3 hr 34 m in 40 sec

24. A 35-year-old man has a 6-year history of weight loss, chronic abdominal pain, bloating, and frequent voluminous, greasy, malodorous stools. He had similar symptoms as an adolescent, but they resolved spontaneously within 2 years. His mother has a similar illness that is controlled by restricting wheat, barley, and rye in her diet. In addition to the gastrointestinal symptoms, the patient has a chronic pruritic rash shown in the photograph of his elbow. Which of the following is the most likely diagnosis of this patient's skin condition? A) Bullous pemphigoid B) Dermatitis herpetiformis C) Epidermolysis bullosa O D) Erythema nodosum O E) Pemphigus vulgaris

r 0

National Board o f Medical Examiners - Google Chrome https://www.starttest.com/api/5.1.1.0/TTDStart.aspx?SVC=f3fd94ad-4a91-4962-b764-2d4caccal061

Exam Section 3: Item 25 o f 50 I Mark

N ational Board o f M edical E xam iners C om prehensive Basic Science Self-Assessment

T im e R em aining: 3 hr 33 m in 36 sec

25. A 1-year-old girl is brought to the physician because of multiple bacterial infections since birth. She is at the 40th percentile for length and 40th percentile for weight. She has white hair, pale skin, blue irises, and prominent red pupils. A complete blood count shows neutropenia. A peripheral smear shows giant granules in the neutrophils. This patient's disorder is most likely due to which of the following pathogenetic mechanisms? O A) Bone marrow suppression
0

B) Defect in leukocyte adhesion C) Defect in phagolysosome function D) Myeloperoxidase deficiency

E) NAD PH oxidase deficiency

N ational Board o f Medical Examiners - Google Chrome Q h ttp s://w w w .sta rtte st.e o m /a p i/5 .l.1.0/TTDStart.aspx?SVC=f3fd94ad-4a91-4962-b764-2d4caccal061

Exam Section 3: Item 26 o f 50 I Mark

National Board o f Medical Examiners Comprehensive Basic Science Self-Assessment

Tim e Remaining: 3 hr 32 min 13 sec

26. A 36-year-old man who is seropositive for HIV has had a nonproductive cough for 2 weeks. An x-ray of the chest shows a diffuse interstitial infiltrate. Silver stain of a lung biopsy specimen is shown. Which of the following is the most likely causal organism? p 1
Previous Lab Values Review Pause
3:07 PM 28/08/2012

r 0

National Board o f Medical Examiners - Google Chrome https://www.starttest.com/api/5.1.1.0/ITDStart.aspx?SVC=f3fd94ad-4a91-4962-b764-2d4caccal061

Exam Section 3: Item 26 o f 50 I Mark

National Board o f Medical Examiners Comprehensive Basic Science Self-Assessment

Tim e Remaining: 3 hr 32 min 6 sec A

26. A 36-year-old man who is seropositive for HIV has had a nonproductive cough for 2 weeks. An x-ray of the chest shows a diffuse interstitial infiltrate. Silver stain of a lung biopsy specimen is shown. Which of the following is the most likely causal organism?

Candida tropicalis B) Coccidioides immitis C) Cryptococcus neoformans


A) D) Cytomegalovirus E)

Mycobacterium tuberculosis O F) Pneumocystis jiroveci (formerly P. carinii)

r 0

National Board o f Medical Examiners - Google Chrome https://www.starttest.com/api/5.1.1.0/TTDStart.aspx?SVC=f3fd94ad-4a91-4962-b764-2d4caccal061

Exam Section 3: Item 27 o f 50 I Mark

N ational Board o f M edical E xam iners C om prehensive Basic Science Self-Assessment

T im e R em aining: 3 hr 31 m in 24 sec

27. A 60-year-old man comes to the physician because of a 2-year history of progressive shortness of breath with exertion. His two younger siblings have had similar symptoms. His mother died of dilated cardiomyopathy. Physical examination shows jugular venous distention and ankle edema. There is hepatomegaly. Inspiratory crackles are heard over both lung bases. A chest x-ray shows cardiomegaly and pulmonary congestion; angiography confirms dilated cardiomyopathy. An atrial endocardial biopsy specimen shows abnormal sarcomeres with paracrystalline inclusions in mitochondria. Sequencing of mitochondrial DNA shows a G-to-A transition. This results in an added A:T base pair to the 3' end of the tRNA, thus shortening the C loop. Which of the following is the most likely effect of this mutation on mitochondrial function? A) Decreased mRNA stability B) Decreased protein synthesis C) Decreased replication D) Increased mRNA stability E) Increased protein synthesis O F) Increased replication

r 0

National Board o f Medical Examiners - Google Chrome https://www.starttest.com/api/5.1.1.0/TTDStart.aspx?SVC=f3fd94ad-4a91-4962-b764-2d4caccal061

Exam Section 3: Item 28 o f 50 I Mark

N ational Board o f M edical E xam iners C om prehensive Basic Science Self-Assessment

T im e R em aining: 3 hr 28 m in 48 sec

28. A 55-year-old man comes to the physician because of intermittent heartburn during the past 3 months. He says that he has approximately two episodes weekly during which he develops burning pain between his chest and abdomen when he lies down for a nap after dinner. A bitter taste sometimes accompanies the pain. He often has an alcoholic drink before dinner and several glasses of wine with the meal. He is 178 cm (5 ft 10 in) tall and weighs 113 kg (250 lb); BMI is 36 kg/m2. Physical examination shows no other abnormalities. In addition to weight loss and decreased alcohol consumption, which of the following drugs is most appropriate for this patient? C A) Loperamide B) Misoprostol C C) Omeprazole C D) Prochlorperazine C E) Sucralfate

r 0

National Board o f Medical Examiners - Google Chrome https://www.starttest.com/api/5.1.1.0/ITDStart.aspx?SVC=f3fd94ad-4a91-4962-b764-2d4caccal061

Exam Section 3: Item 29 o f 50 I Mark

N ational Board o f M edical E xam iners C om prehensive Basic Science Self-Assessment

T im e R em aining: 3 hr 27 m in 59 sec

29. A 35-year-old man comes to the physician because of a chronic cough for 5 months. He says, "I worry I may have lung cancer like my dad." He has smoked 2 packs of cigarettes daily for 15 years. Physical examination and an x-ray of the chest show no abnormalities. After the physician informs the patient of the findings, which of the following responses is most appropriate to encourage this patient to quit smoking? A) "Have you ever considered changing to chewing tobacco?" B) "Have you ever thought about quitting smoking?" C) "I'd like to refer you to a hypnotist to help you quit smoking." O D) "I'll prescribe some nicotine patches for you. That will help you cut down on your smoking." E) "Try changing to low-nicotine cigarettes for a while. That will decrease your chances of getting lung cancer." F) "You must stop smoking immediately, or you will most likely get lung cancer someday like your father."

r 0

National Board o f Medical Examiners - Google Chrome https://www.starttest.com/api/5.1.1.0/TTDStart.aspx?SVC=f3fd94ad-4a91-4962-b764-2d4caccal061

Exam Section 3: Item 30 o f 50 I Mark

N ational Board o f M edical E xam iners C om prehensive Basic Science Self-Assessment

T im e R em aining: 3 hr 27 m in 10 sec

30. A 22-year-old man comes to the physician because of a 2-year history of episodes of shortness of breath and wheezing. His respirations are 12/min. Physical examination shows no abnormalities. It is found that the symptoms occur after he takes certain over-the-counter medications containing aspirin. Laboratory studies show a serum IgE concentration within the reference range and no eosinophilia. This patient's symptoms are most likely caused by excessive production of which of the following substances? O A) Cyclooxygenase B) Hydroxyeicosatetraenoic acid O C) Leukotrienes O D) Prostacyclin (PGI2) E) Prostaglandins F) Thromboxane A 2

r 0

National Board o f Medical Examiners - Google Chrome https://www.starttest.com/api/5.1.1.0/TTDStart.aspx?SVC=f3fd94ad-4a91-4962-b764-2d4caccal061

Exam Section 3: Item 31 o f 50 I Mark

N ational Board o f M edical E xam iners C om prehensive Basic Science Self-Assessment

T im e R em aining: 3 hr 25 m in 32 sec

31.

A 73-year-old woman is brought to the physician because of a 1-month history of urinary frequency and urgency. She also has had a 3-month history of difficulties with walking, conversation, and memory. Neurologic examination shows normal strength and sensation. Her Mini-Mental State Examination score is 21/30. She walks with a wide-based, shuffling gait. A CT scan of the head shows marked enlargement of the ventricles. Which of the following is the most likely diagnosis? A) Atherosclerosis of the carotid artery

O B) Dementia, Alzheimer type O C) Huntington disease o D) Normal-pressure hydrocephalus E) Parkinson disease F) Small vessel disease

r 0

National Board o f Medical Examiners - Google Chrome https://www.starttest.com/api/5.1.1.0/ITDStart.aspx?SVC=f3fd94ad-4a91-4962-b764-2d4caccal061

Exam Section 3: Item 32 o f 50 I Mark

N ational Board o f M edical E xam iners C om prehensive Basic Science Self-Assessment

T im e R em aining: 3 hr 25 m in 22 sec

32.

In order to determine risk factors for death from coronary artery disease (CAD), persons aged 30 years and older are studied in a community. Every 2 years for a total of 10 years, blood pressure, serum cholesterol concentration, and smoking status are assessed along with deaths from CAD. Persons with increased blood pressure are compared with those who have normal blood pressure to determine which group has greater mortality from coronary artery disease. Which of the following best describes this type of study?

O A) Case-control study O B) Case series study C) Cross-sectional study O D) Ecologic study E) Observational cohort study F) Randomized clinical trial

v---------- 1----------l------------------' -

0 https:/7www.starttest.com/api/5.1.1.0/ITDStart.aspx?SVC=f3fd94ad-4a91-4962-b764-2d4caccal061 Exam Section 3: Item 33 o f 50 H Mark National Board o f Medical Examiners Comprehensive Basic Science Self-Assessment Tim e Remaining: 3 hr 23 min 28 sec

33. A 33-year-old woman comes to the physician 3 days after she found a mass in her right breast on self-examination. Examination of the right breast shows a 2.5-cm, hard mass in the upper outer quadrant of the right breast. A photograph of the resected mass is shown. Which of the following characteristics of the lesion best predicts this patient's prognosis? A) Degree of differentiation of the cells comprising the lesion B) DNA content of the cells comprising the lesion C) Lymph node involvement by cells originating in the lesion D) Presence or absence of hormone receptors within the lesion E) Size of the lesion

Previous

p 1
Lab Values

Review

Pause 3:16 PM 28/08/2012

Cl

r 0

National Board o f Medical Examiners - Google Chrome https://www.starttest.com/api/5.1.1.0/TTDStart.aspx?SVC=f3fd94ad-4a91-4962-b764-2d4caccal061

Exam Section 3: Item 34 o f 50 I Mark

N ational Board o f M edical E xam iners C om prehensive Basic Science Self-Assessment

T im e R em aining: 3 hr 21 m in 5 sec

34. A 49-year-old man is brought to the emergency department 30 minutes after fainting in the street. He regained consciousness 1 minute after fainting. He says that he has had watery diarrhea during the past 5 days, which has not improved with fasting. He has not changed his diet or travelled overseas recently. His blood pressure is 90/60 mm Hg. Physical examination shows a flushed face and dehydration. Serum studies show a potassium concentration of 2 mEq/L and glucose concentration of 150 mg/dL. He is admitted to the hospital, and intravenous fluid replacement is started. Over the next day, he passes a stool with a volume of 3.5 L. Which of the following is the most likely cause of this patient's diarrhea? A) Bacterial overgrowth B) Cholera C) Inactivation of lipase D) Lactose intolerance E) Vasoactive intestinal polypeptide secretion

r 0

National Board o f Medical Examiners - Google Chrome https://www.starttest.com/api/5.1.1.0/ITDStart.aspx?SVC=f3fd94ad-4a91-4962-b764-2d4caccal061

Exam Section 3: Item 35 o f 50 I Mark

N ational Board o f M edical E xam iners C om prehensive Basic Science Self-Assessment

T im e R em aining: 3 hr 20 m in 52 sec

35.

A 76-year-old man with congestive heart failure comes to the physician because of a 1-month history of shortness of breath after walking up a flight of stairs. He has had frequent nausea, and he has had to use two pillows to sleep during this period. Diffuse, moist crackles are heard over both lungs. There is pitting edema above the ankles. Which of the following best describes the function of the product secreted by the atrial myocytes in this patient? A) Decreased glomerular filtration rate B) Increased sodium reabsorption

O C) Induction of vasoconstriction O D) Inhibition of renin release E) Stimulation of aldosterone release

r 0

National Board o f Medical Examiners - Google Chrome https://www.starttest.com/api/5.1.1.0/TTDStart.aspx?SVC=f3fd94ad-4a91-4962-b764-2d4caccal061

Exam Section 3: Item 36 o f 50 I Mark

N ational Board o f M edical E xam iners C om prehensive Basic Science Self-Assessment

T im e R em aining: 3 hr 19 m in 38 sec

36. A 19-year-old woman comes to the physician because of increasingly severe pain in the right hip during the past month. She has a 4-year history of refractory inflammatory bowel disease with arthritis in the hands treated with corticosteroids for the past 2 years. An x-ray of the pelvis shows collapse of the superior half of the right femoral head with preservation of the articular cartilage. The left hip appears normal. Which of the following is the most likely cause of the hip abnormality? A) Ankylosing spondylitis

O B) Aseptic necrosis O C) Gouty arthritis D) Multiple myeloma O E) Septic arthritis

r 0

National Board o f Medical Examiners - Google Chrome https://www.starttest.com/api/5.1.1.0/TTDStart.aspx?SVC=f3fd94ad-4a91-4962-b764-2d4caccal061

Exam Section 3: Item 37 o f 50 I Mark

N ational Board o f M edical E xam iners C om prehensive Basic Science Self-Assessment

T im e R em aining: 3 hr 17 m in 16 sec

37. An executor of an estate consults with a physician concerning the terms of a will. The deceased woman was a philanthropist who was active in addressing disparities in health care. She designated that a large sum of her money be used to educate the public about the leading cause of death in women. The most appropriate use of this money would be a program addressing which of the following diseases? O A) Breast cancer B) Cardiovascular disease C) Cerebral infarction D) Cervical cancer E) Ovarian cancer

r 0

National Board o f Medical Examiners - Google Chrome https://www.starttest.com/api/5.1.1.0/ITDStart.aspx?SVC=f3fd94ad-4a91-4962-b764-2d4caccal061

Exam Section 3: Item 38 o f 50 I Mark

N ational Board o f M edical E xam iners C om prehensive Basic Science Self-Assessment

T im e R em aining: 3 hr 14 m in 50 sec

38. A 2-year-old girl is brought to the physician by her mother because of fever and a sore throat for 3 days. She has difficulty swallowing and has become irritable, listless, and uninterested in eating. She was recently adopted from a developing country, and her immunization history is unknown. Her temperature is 38.5C (101.3F), pulse is 150/min, and respirations are 40/min. Physical examination shows a dull red throat. A gray exudate is present on the tonsils, pharynx, uvula, and tongue, and the exudate can be lifted with a tongue blade. There is marked cervical lymphadenopathy. The most likely causal organism produces a toxin that causes ADP-ribosylation of which of the following targets? A) Acetylcholinesterase

C B) Adenylyl cyclase C C) Elongation factor-2 D) GTPase Gs subunit C E) Guanylyl cyclase O F) Ribosomal protein S1

r 0

National Board o f Medical Examiners - Google Chrome https://www.starttest.com/api/5.1.1.0/TTDStart.aspx?SVC=f3fd94ad-4a91-4962-b764-2d4caccal061

Exam Section 3: Item 39 o f 50 I Mark

N ational Board o f M edical E xam iners C om prehensive Basic Science Self-Assessment

T im e R em aining: 3 hr 13 m in 34 sec

39. A 70-year-old woman comes to the physician because of several episodes of burning pain with urination during the past year. Physical examination shows uterine prolapse. Urinalysis shows a mild increase in leukocytes. Ultrasonography of the abdomen shows bilateral dilation of the renal collecting system. This patient is at greatest risk for which of the following? O A) Cervical carcinoma B) Pyelonephritis C) Renal cell carcinoma D) Ureterolithiasis E) Urothelial carcinoma

National Board o f Medical Examiners - Google Chrome Q h ttp s://w w w .sta rtte st.e o m /a p i/5 .l.1.0/TTDStart.aspx?SVC=f3fd94ad-4a91-4962-b764-2d4caccal061

1 cz, I IB 1 ^ 3 - 1

Exam Section 3: Item 40 o f 50 M ark

N ational Board o f M edical E xam iners C om prehensive Basic Science Self-Assessment

T im e R em aining: 3 hr 12 m in 15 sec

40.

A 6-week-old male newborn has persistent nonbilious projectile vomiting. Physical examination shows prominent peristalsis. Laboratory findings are indicative of a metabolic alkalosis. Which of the following is the most likely mechanism of this condition? A) Aganglionosis of the large bowel B) Congenital atresia of the bile ducts C) Congenital defect in the diaphragm

O D) Hypertrophy of the pyloric sphincter O E) Intestinal obstruction by meconium

r 0

National Board o f Medical Examiners - Google Chrome https://www.starttest.com/api/5.1.1.0/ITDStart.aspx?SVC=f3fd94ad-4a91-4962-b764-2d4caccal061

Exam Section 3: Item 41 o f 50 I Mark

N ational Board o f M edical E xam iners C om prehensive Basic Science Self-Assessment

T im e R em aining: 3 hr 12 m in 13 sec

41. A 32-year-old man has had generalized band-like headaches and dizziness almost every day for the past 6 months. Physical examination and CT scan of the head show no abnormalities. Which of the following additional findings in this patient is consistent with a diagnosis of hypochondriasis rather than undifferentiated somatoform disorder? A) Disproportionate fear of serious disease B) Frequent health care visits C) Past history of unexplained symptoms D) Persistence of symptoms longer than 6 months O E) Presence of depressive symptoms

r 0

National Board o f Medical Examiners - Google Chrome https://www.starttest.com/api/5.1.1.0/TTDStart.aspx?SVC=f3fd94ad-4a91-4962-b764-2d4caccal061

Exam Section 3: Item 42 o f 50 I Mark

N ational Board o f M edical E xam iners C om prehensive Basic Science Self-Assessment

T im e R em aining: 3 hr 12 m in 10 sec

42. A 22-year-old nulligravid woman develops right flank pain 3 days after undergoing an appendectomy and right ovarian resection because of an appendiceal abscess. Menses occur at regular 28-day intervals. Vital signs are within normal limits. Physical examination shows mild right flank tenderness. The most likely cause of this patient's pain is the accidental ligation of which of the following structures on the right? A) Ovarian artery

B) Ovarian vein O C) Renal artery D) Renal vein E) Ureter F) Uterine ligament

r 0

National Board o f Medical Examiners - Google Chrome https://www.starttest.com/api/5.1.1.0/TTDStart.aspx?SVC=f3fd94ad-4a91-4962-b764-2d4caccal061

Exam Section 3: Item 43 o f 50 I Mark

N ational Board o f M edical E xam iners C om prehensive Basic Science Self-Assessment

T im e R em aining: 3 hr 12 m in 7 sec

43. A 5-year-old boy is brought to the physician by his mother because of progressive clumsiness and fatigue during the past 6 months. He says that his legs are tired. He was delivered at term after an uncomplicated pregnancy. He has met all developmental milestones, although there was some delay compared with other children his age. He is alert. He has difficulty rising from the chair; he uses his arms to push himself into a standing position. He is unable to jump with both feet together. Physical examination shows hypertrophy of the calf muscles. This patient most likely has weak hip adduction as a result of dysfunction of the muscle inserting onto the femur from which of the following locations? A) Anterior sacrum B) Iliac crest O C) Iliac spine D) Ischium E) Lateral ilium

r 0

National Board o f Medical Examiners - Google Chrome https://www.starttest.com/api/5.1.1.0/ITDStart.aspx?SVC=f3fd94ad-4a91-4962-b764-2d4caccal061

Exam Section 3: Item 44 o f 50 I Mark

N ational Board o f M edical E xam iners C om prehensive Basic Science Self-Assessment

T im e R em aining: 3 hr 12 m in 4 sec

44. A 35-year-old man who is a farmer is brought to the hospital by his wife because of difficulty breathing, sweating, excess salivation, and diarrhea for the past 2 hours. His wife says that the symptoms started when he was applying a new insecticide to his crops. Temperature is 36C (96.8F), pulse is 50/min, respirations are 22/min, and blood pressure is 90/60 mm Hg. Which of the following is the most appropriate treatment? O A) Atropine B) Ipratropium C) Mecamylamine D) Neostigmine O E) Propantheline

r 0

National Board o f Medical Examiners - Google Chrome https://www.starttest.com/api/5.1.1.0/TTDStart.aspx?SVC=f3fd94ad-4a91-4962-b764-2d4caccal061

Exam Section 3: Item 45 o f 50 I Mark

N ational Board o f M edical E xam iners C om prehensive Basic Science Self-Assessment

T im e R em aining: 3 hr 12 m in 1 sec

45. A 59-year-old woman with metastatic renal carcinoma is receiving cyclophosphamide, paclitaxel, and interleukin-2 (IL-2). Which of the following is the reason for including IL-2 in the regimen? A) Decreasing bone marrow suppression B) Decreasing the likelihood of hemorrhagic cystitis C) Decreasing the likelihood of peripheral neuropathy D) Decreasing nausea and vomiting O E)Stim ulating!lym phocytes

r 0

National Board o f Medical Examiners - Google Chrome https://www.starttest.com/api/5.1.1.0/TTDStart.aspx?SVC=f3fd94ad-4a91-4962-b764-2d4caccal061

Exam Section 3: Item 46 o f 50 I Mark

N ational Board o f M edical E xam iners C om prehensive Basic Science Self-Assessment

T im e R em aining: 3 hr 11 m in 58 sec

46. An investigator is studying (32-adrenoreceptors in female experimental animals. During the experiment, epinephrine is injected intramuscularly into each animal, and the effects on (32 adrenoreceptors are then observed. Which of the following physiologic effects is most likely to be observed in these animals? A) Increased myocardial contractility B) Internal urethral sphincter contraction C) Lipolysis D) Pilomotor contraction E) Pupillary dilation F) Uterine relaxation

r 0

National Board o f Medical Examiners - Google Chrome https://www.starttest.com/api/5.1.1.0/ITDStart.aspx?SVC=f3fd94ad-4a91-4962-b764-2d4caccal061

Exam Section 3: Item 47 o f 50 I Mark

N ational Board o f M edical E xam iners C om prehensive Basic Science Self-Assessment

T im e R em aining: 3 hr 11 m in 55 sec

47. A 5-month-old boy is brought to the physician because of a 5-day history of difficulty feeding. Physical examination shows macroglossia and enlargement of the anterior neck at the midline. A CT scan of the neck shows a 4-cm, well-defined, cystic structure. A remnant of which of the following is the most likely cause of this patient's condition? O A) Cervical sinus O B) First pharyngeal pouch O C) Fourth pharyngeal pouch O D) Occipital myotome O E) Thyroglossal duct

r 0

National Board o f Medical Examiners - Google Chrome https://www.starttest.com/api/5.1.1.0/TTDStart.aspx?SVC=f3fd94ad-4a91-4962-b764-2d4caccal061

Exam Section 3: Item 48 o f 50 I Mark

N ational Board o f M edical E xam iners C om prehensive Basic Science Self-Assessment

T im e R em aining: 3 hr 11 m in 52 sec

48.

An 8-year-old boy with severe mental retardation is brought to the physician because of a 1-year history of self-mutilating behavior, including biting his fingers. His male cousin has similar symptoms. Physical examination shows spasticity, chorea, and mutilation of the digits. Laboratory studies are most likely to show which of the following abnormalities?

O A) Hypercalcemia O B) Hypercalciuria O C) Hyperuricemia D) Hypokalemia O E) Hyponatremia F)Hypophosphatemia

r 0

National Board o f Medical Examiners - Google Chrome https://www.starttest.com/api/5.1.1.0/TTDStart.aspx?SVC=f3fd94ad-4a91-4962-b764-2d4caccal061

Exam Section 3: Item 49 o f 50 I Mark

N ational Board o f M edical E xam iners C om prehensive Basic Science Self-Assessment

T im e R em aining: 3 hr 11 m in 48 sec

49.

A 76-year-old woman who is a resident of a skilled nursing care facility is brought to the physician because of a 2-day history of fever and cough. Her temperature is 38.5C (101.3F). Treatment with ciprofloxacin is begun. Two days later, she is admitted to the hospital because of a progressive cough and a temperature of 39C (102.2F). Culture of sputum at this time grows Streptococcus pneumoniae. A mutation in genes encoding which of the following enzymes is the most likely cause of the organism's resistance to ciprofloxacin? A) Acetyltransferase

O B) DNA gyrase O C) 3-Lactamase D) Tetrahydrofolate reductase E) Transpeptidase

N a tio n a l

Board of Medical Examiners - Google Chrome

i =

,@

Q h ttp s://w w w .sta rtte st.e o m /a p i/5 .l.1.0/TTDStart.aspx?SVC=f3fd94ad-4a91-4962-b764-2d4caccal061

Exam Section 3: Item 50 o f 50 H Mark

National Board o f Medical Examiners Comprehensive Basic Science Self-Assessment

Tim e Remaining: 3 hr 11 min 38 sec

50. A 54-year-old man has a squamous cell carcinoma of the right lung. Analysis of his germline DNA shows two alleles, m and n, at the microsatellite locus Z tightly linked to the p53 gene. Similar analysis of the neoplastic cells shows the presence of the m allele only. These findings are most likely due to which of the following processes? O A) Germline mosaicism O B) Loss of heterozygosity C) p53 Gene amplification O D) Pericentric inversion involving p53 and locus Z O E) Point mutation of allele n

r 0

National Board o f Medical Examiners - Google Chrome https://www.starttest.com/api/5.1.1.0/TTDStart.aspx?SVC=f3fd94ad-4a91-4962-b764-2d4caccal061

Exam Section 4: Item 1 o f 50 I Mark

N ational Board o f M edical E xam iners C om prehensive Basic Science Self-Assessment

T im e R em aining: 4 hr 19 m in 19 sec

1. A 70-year-old man comes to the emergency department 30 minutes after the onset of weakness, nausea, and lower abdominal and back pain that radiates to the lower extremities. He appears pale and has a cold sweat. Pulse is 110/min and blood pressure is 90/60 mm Hg. Pulses in the lower extremities are weak. A faint bruit is heard in the lower abdomen. Muscle stretch reflexes are normal in both lower extremities. Which of the following is the most likely diagnosis? A)
0

Acute compression fracture of the L-3 vertebral body

B) Herniated L-3 to L-4 intervertebral disc C) Leaking aortic aneurysm D) Leaking left renal artery aneurysm

E) Perforated rectum

O F) Perforated sigmoid colon

r 0

National Board o f Medical Examiners - Google Chrome https://www.starttest.com/api/5.1.1.0/TTDStart.aspx?SVC=f3fd94ad-4a91-4962-b764-2d4caccal061

Exam Section 4: Item 2 o f 50 I Mark

N ational Board o f M edical E xam iners C om prehensive Basic Science Self-Assessment

T im e R em aining: 4 hr 19 m in 10 sec

2. A 45-year-old man is brought to the emergency department because of a 6-hour history of severe headache and confusion. He says that it is the worst headache he has ever had. He appears lethargic. His blood pressure is 130/80 mm Hg. Physical examination shows nuchal rigidity. A CT scan of the head without contrast is shown. Which of the following is the most likely cause of this patient's condition? A) Diffuse cerebral edema

O B) Hydrocephalus O C) Large mass in the posterior fossa D) Subarachnoid hemorrhage E) Subdural hematoma

r 0

National Board o f Medical Examiners - Google Chrome https://www.starttest.com/api/5.1.1.0/TTDStart.aspx?SVC=f3fd94ad-4a91-4962-b764-2d4caccal061

Exam Section 4: Item 3 o f 50 I Mark

N ational Board o f M edical E xam iners C om prehensive Basic Science Self-Assessment

T im e R em aining: 4 hr 18 m in 55 sec

3. An otherwise healthy 23-year-old woman comes to the physician because she has not had a menstrual period for 2 months. Menarche occurred at the age of 14 years, and menses had occurred at regular 28-day intervals. Pelvic examination shows blue-tinged vaginal mucosa. The uterus is somewhat soft in the lower segment and is about the size of an orange. The adnexa are nontender with normal-sized ovaries. Which of the following is the most appropriate next step to establish the diagnosis? A) Measurement of serum follicle-stimulating hormone concentration B) Measurement of serum thyroid-stimulating hormone concentration C) Measurement of urine 3-human chorionic gonadotropin concentration D) Ultrasonography of the pelvis O E) Endometrial biopsy

National Board o f Medical Examiners - Google Chrome Q https:/?www.starttest.com/api/5.1.1.0/TTDStart.aspx?SVC=f3fd94ad-4a91-4962-b764-2d4caccal061

Exam Section 4: Item 4 o f 50 I Mark

N ational Board o f M edical E xam iners C om prehensive Basic Science Self-Assessment

T im e R em aining: 4 hr 18 m in 53 sec

4. A 25-year-old man is diagnosed with advanced testicular cancer. Combination chemotherapy is being considered. Cisplatin is included in the regimen because of its effect on which of the following? A) Dihydrofolate reductase

B) DNA O C) Microtubules D) Purine ring biosynthesis E) Ribonucleotide reductase

Previous

Lab Values

Review

&
Help

Pause

O
3:53 PM 28/08/2012

I)

&

rSTEaI

rfcjrxy' r^i

r 0

National Board o f Medical Examiners - Google Chrome https://www.starttest.com/api/5.1.1.0/ITDStart.aspx?SVC=f3fd94ad-4a91-4962-b764-2d4caccal061

Exam Section 4: Item 5 o f 50 I Mark

N ational Board o f M edical E xam iners C om prehensive Basic Science Self-Assessment

T im e R em aining: 4 hr 18 m in 34 sec

5. A 72-year-old man is brought to the emergency department because of a 1-week history of increasingly severe abdominal pain, diarrhea, and loss of appetite. He has had a fever for 2 days. His temperature is 38.3C (101 F), and pulse is 106/min. Abdominal examination shows distention and rebound tenderness localized to the left lower quadrant. His leukocyte count is 18,000/mm3 (80% segmented neutrophils, 10% bands, and 10% lymphocytes). An abdominal CT scan with contrast shows a 7-mm, thickened colonic wall, and pericolic fat inflammation; there is a collection of contrast in this region. Which of the following is the most likely diagnosis? O A) Amebiasis O B) Collagenous colitis C) Colon cancer D) Diverticulitis E) Ulcerative colitis

r 0

National Board o f Medical Examiners - Google Chrome https://www.starttest.com/api/5.1.1.0/TTDStart.aspx?SVC=f3fd94ad-4a91-4962-b764-2d4caccal061

Exam Section 4: Item 6 o f 50 I Mark

N ational Board o f M edical E xam iners C om prehensive Basic Science Self-Assessment

T im e R em aining: 4 hr 18 m in 31 sec

6. A 6-year-old boy is found to have almond-shaped eyes, temperature instability, mental retardation, hyperphagia, and obesity. Cytogenetic analysis shows a balanced translocation involving chromosome 15q11-q13. His father and paternal aunt are found to have an identical balanced translocation. His aunt's 5-year-old daughter has a history of inappropriate laughter, abnormal gait, and mental retardation. Which of the following genetic principles best explains the findings in this family? O A) Anticipation 0 B) Genetic drift

0 C) Heteroplasmy
D) Imprinting 0 E) Pseudodominant inheritance

r 0

National Board o f Medical Examiners - Google Chrome https://www.starttest.com/api/5.1.1.0/TTDStart.aspx?SVC=f3fd94ad-4a91-4962-b764-2d4caccal061

Exam Section 4: Item 7 o f 50 I Mark

N ational Board o f M edical E xam iners C om prehensive Basic Science Self-Assessment

T im e R em aining: 4 hr 18 m in 28 sec

7. A 31 -year-old woman is brought to the emergency department by her husband because of difficulty breathing and severe muscle weakness for 10 minutes. She has a 3-year history of myasthenia gravis treated with neostigmine. The husband reports that she doubled her dosage 2 days ago because she was feeling extraordinarily weak, but her weakness has increased since then. Which of the following events is the most likely cause of the increased muscle weakness in this patient? O A) Autoimmune hyperactivation of nicotinic receptors B) Autoimmune inactivation of muscarinic receptors C) Desensitization of nicotinic receptors D) Excessive degradation of acetylcholine E) Hypersensitization of muscarinic receptors F) Insufficient release of acetylcholine

r 0

National Board o f Medical Examiners - Google Chrome https://www.starttest.com/api/5.1.1.0/ITDStart.aspx?SVC=f3fd94ad-4a91-4962-b764-2d4caccal061

Exam Section 4: Item 8 o f 50 I Mark

N ational Board o f M edical E xam iners C om prehensive Basic Science Self-Assessment

T im e R em aining: 4 hr 18 m in 25 sec

8. A 2-year-old girl takes oral cephalexin for treatment of impetigo caused by group A (3-hemolytic patient? A) Binding to bacterial 30S ribosomal subunits B) Binding to bacterial 50S ribosomal subunits C) Inhibition of bacterial dihydrofolate reductase D) Interference with bacterial cell wall synthesis E) Production of 3-lactamase

Streptococcus. Which of the following is the mechanism of action of this antibiotic in this

r 0

National Board o f Medical Examiners - Google Chrome https://www.starttest.com/api/5.1.1.0/TTDStart.aspx?SVC=f3fd94ad-4a91-4962-b764-2d4caccal061

Exam Section 4: Item 9 o f 50 I Mark

N ational Board o f M edical E xam iners C om prehensive Basic Science Self-Assessment

T im e R em aining: 4 hr 18 m in 22 sec

9. A previously healthy 42-year-old Asian woman is brought to the emergency department because of a 24-hour history of nausea, vomiting, and progressive lethargy. She has smoked 1 pack of cigarettes daily for 25 years and drinks four glasses of wine daily. She uses high-dose acetaminophen daily for headaches. She does not use illicit drugs. She is 155 cm (5 ft 1 in) tall and weighs 50 kg (110 lb); BMI is 21 kg/m2. She is responsive to painful stimuli. Initial laboratory studies show increased hepatic aminotransferase. Which of the following effects of alcohol most likely contributed to this patient's condition? A) Decreased generation of/V-acetyl-p-benzoquinoneimine B) Increased glucuronidation C) Increased hepatic glutathione stores D) Increased sulfation O E) Induction of cytochrome P450

r 0

National Board o f Medical Examiners - Google Chrome https://www.starttest.com/api/5.1.1.0/TTDStart.aspx?SVC=f3fd94ad-4a91-4962-b764-2d4caccal061

Exam Section 4: Item 10 o f 50 I Mark

N ational Board o f M edical E xam iners C om prehensive Basic Science Self-Assessment

T im e R em aining: 4 hr 18 m in 19 sec

10. A prospective study examined the relationship between the development of gastric cancer and exposure to a diet rich in selenium. The investigators estimated the relative risk of gastric cancer to be 0.3 in individuals with a high-selenium diet (95% confidence interval 0.1-0.8). Which of the following is the most appropriate interpretation of this finding? A) Selenium causes gastric cancer B) Selenium exposure is unrelated to gastric cancer C) Selenium is associated with a higher risk of gastric cancer D) Selenium is associated with a lower risk of gastric cancer E) Selenium is positively correlated with gastric cancer

r~'

National Board o f Medical Examiners - Google Chrome https:/?www.starttest.com/api/5.1.1.0/ITDStart.aspx?SVC=a2401610-d3a2-45ec-83a7-b56dlda82cf6

I1a

\ < @

Exam Section 4: Item 11 o f 50 I Mark

N ational Board o f M edical E xam iners C om prehensive Basic Science Self-Assessment

T im e R em aining: 4 hr 18 m in 1 sec

11. A 78-year-old man comes to the physician because of a 3-month history of increasingly severe back pain. He has had a 5.4-kg (12-lb) weight loss during this period. The pain occurs mostly at night, is exacerbated by movement, and is no longer relieved by aspirin. There has been no trauma to the area. He appears chronically ill. His pulse is 100/min, and blood pressure is 160/100 mm Hg. Physical examination shows point tenderness over the L4-5 area; there are no skin lesions. Urinalysis shows 4+ protein. A photomicrograph of findings from bone marrow aspiration are shown. Which of the following is the most likely diagnosis? A) Hyperparathyroidism B) Metastatic bone disease C) Multiple myeloma D) Osteitis deformans (Paget disease) E) Waldenstrom macroglobulinemia

r~'

National Board o f Medical Examiners - Google Chrome https:/7www.starttest.com/api/5.1.1.0/ITDStart.aspx?SVC=a2401610-d3a2-45ec-83a7-b56dlda82cf6

F z* m

Exam Section 4: Item 12 o f 50 I Mark

N ational Board o f M edical E xam iners C om prehensive Basic Science Self-Assessment

T im e R em aining: 4 hr 17 m in 59 sec

12. A 32-year-old woman comes to the physician because she has not been able to conceive for 5 years. Previous fertility examinations of her husband showed no abnormalities. Menses occur at regular 28-day intervals. Her blood pressure is 110/70 mm Hg. Physical and pelvic examinations show no abnormalities. Hysterosalpingography shows normal findings. Her serum hormone concentrations are within the reference range. Which of the following is the most appropriate pharmacotherapy for this patient's infertility? O A) Bromocriptine O B)Clomiphene C) DiethylstiIbestrol D) Ethinyl estradiol E) Medroxyprogesterone

r~'

National Board o f Medical Examiners - Google Chrome https:/7www.starttest.com/api/5.1.1.0/ITDStart.aspx?SVC=a2401610-d3a2-45ec-83a7-b56dlda82cf6

F z* m

Exam Section 4: Item 13 o f 50 I Mark

N ational Board o f M edical E xam iners C om prehensive Basic Science Self-Assessment

T im e R em aining: 4 hr 17 m in 57 sec

13. A 70-year-old man comes to the physician because of a rash for 7 days. Physical examination shows multiple 4- to 6-cm tense bullae filled with clear fluid over the inner aspects of the forearms, lower abdomen, groin, and thighs. Examination of a skin biopsy specimen shows a non-acantholytic blister with separation of the epidermis from the basement membrane. Serologic studies are most likely to show an autoantibody directed against which of the following? A) C) Anchoring filaments Desmosomal proteins

O B) Collagen, type IV O D) Hemidesmosomal proteins E) Intracellular adhesion molecule-1

r~'

National Board o f Medical Examiners - Google Chrome https:/7www.starttest.com/api/5.1.1.0/ITDStart.aspx?SVC=a2401610-d3a2-45ec-83a7-b56dlda82cf6

F z* m

Exam Section 4: Item 14 o f 50 I Mark

N ational Board o f M edical E xam iners C om prehensive Basic Science Self-Assessment

T im e R em aining: 4 hr 17 m in 53 sec

14. A 20-year-old man has had hemoptysis, hematuria, and a 10-kg (22-lb) weight loss over the past 3 months. Laboratory studies show a hypochromic microcytic anemia. Which of the following sets of microscopic findings is most likely in tissue obtained on biopsy of his kidney?

Light Microscopy
A) B) C) D) Diffusely hypercellular glomeruli Glomerular crescents Normal glomerular cellularity Normal glomerular cellularity

Deposition in Glomerular Basement Membrane


granular IgG and C3 linear IgG and C3 granular IgG and C3 negative

Electron Microscopy
subepithelial humps no deposits subepithelial deposits loss of foot processes

r~'

National Board o f Medical Examiners - Google Chrome https:/?www.starttest.com/api/5.1.1.0/ITDStart.aspx?SVC=a2401610-d3a2-45ec-83a7-b56dlda82cf6

F z* m

Exam Section 4: Item 15 o f 50 I Mark

N ational Board o f M edical E xam iners C om prehensive Basic Science Self-Assessment

T im e R em aining: 4 hr 17 m in 49 sec

15. A 23-year-old woman with type 1 diabetes mellitus is brought to the physician for a follow-up examination. She has a 2-year history of recurrent yeast infections. Genetic analysis shows a deficiency of myeloperoxidase. Which of the following is the most likely cause of the increased susceptibility to infections in this patient? A) Decreased oxygen consumption after phagocytosis B) Defective neutrophil degranulation C) Defective production of prostaglandins D) Inability to produce hydrogen peroxide E) Inability to produce hydroxy-halide radicals

r~'

National Board o f Medical Examiners - Google Chrome https:/7www.starttest.com/api/5.1.1.0/ITDStart.aspx?SVC=a2401610-d3a2-45ec-83a7-b56dlda82cf6

F z* m

Exam Section 4: Item 16 o f 50 I Mark

N ational Board o f M edical E xam iners C om prehensive Basic Science Self-Assessment

T im e R em aining: 4 hr 0 m in 50 sec

16. A 40-year-old man comes to the physician because of a 6-month history of difficulty maintaining an erection during sexual intercourse. He has consumed 1 L of bourbon daily for /4 15 years. His pulse is 88/min, and blood pressure is 130/80 mm Hg. Examination shows scleral icterus and spider angiomata over the trunk. The liver span is 5 to 6 cm in the midclavicular line. The spleen tip is palpated 5 to 6 cm below the left costal margin. Decreased serum concentrations of which of the following is the most likely cause of this patient's erectile dysfunction? O A) Estradiol
0 0 0 0 0

B) Estrone C) Follicle-stimulating hormone D) Human chorionic gonadotropin E) Luteinizing hormone F) Testosterone

r~'

National Board o f Medical Examiners - Google Chrome https:/7www.starttest.com/api/5.1.1.0/ITDStart.aspx?SVC=a2401610-d3a2-45ec-83a7-b56dlda82cf6

F z* m

Exam Section 4: Item 17 o f 50 I Mark

N ational Board o f M edical E xam iners C om prehensive Basic Science Self-Assessment

T im e R em aining: 3 hr 59 m in 43 sec

17. During an experiment, Drug X is administered to treat trematode and cestode infestations. Results show that Drug X increases the permeability of the cell membranes to calcium, causing paralysis, dislodgement, and death of the parasite. Drug X most closely resembles which of the following? O A) Albendazole O B) Bithionol C) Diethylcarbamazine O D) Niclosamide O E) Praziquantel

r~'

National Board o f Medical Examiners - Google Chrome https:/?www.starttest.com/api/5.1.1.0/ITDStart.aspx?SVC=a2401610-d3a2-45ec-83a7-b56dlda82cf6

F z* m

Exam Section 4: Item 18 o f 50 I Mark

N ational Board o f M edical E xam iners C om prehensive Basic Science Self-Assessment

T im e R em aining: 3 hr 59 m in 24 sec

18. A 25-year-old woman undergoes a radical mastectomy with removal of the axillary lymph nodes on the left side. After the operation, she develops painful edema of the left upper extremity. Which of the following therapies is most likely to decrease the swelling in this patient? O A) Application of heat O B) Compression sleeve O C) Diuretic therapy O D) Exercise therapy O E) Passive movement

r~'

National Board o f Medical Examiners - Google Chrome https:/7www.starttest.com/api/5.1.1.0/ITDStart.aspx?SVC=a2401610-d3a2-45ec-83a7-b56dlda82cf6

F z* m

Exam Section 4: Item 19 o f 50 I Mark

N ational Board o f M edical E xam iners C om prehensive Basic Science Self-Assessment

T im e R em aining: 3 hr 59 m in 15 sec

19. A young adult couple has been unable to conceive for the past 2 years. The woman has regular menstrual cycles and takes no contraceptives. A semen analysis shows: Quality Quantity Color Fructose normal range normal range white 5% of normal content

The most likely cause of this couple's infertility is deficient activity of which of the following in the man? A) Adenohypophysis B) Bulbourethral glands C) Prostate D) Seminal vesicles E) Testes

r 0

National Board o f Medical Examiners - Google Chrome https:/..'www.starttest.com/api/5.1.1.0/ITDStart.aspx?SVC=a2401610-d3a2-45ec-83a7-b56dlda82cf6

Exam Section 4: Item 20 o f 50 I Mark

N ational Board o f M edical E xam iners C om prehensive Basic Science Self-Assessment

T im e R em aining: 3 hr 56 m in 21 sec

20. A cluster of three cases of nosocomial pneumonia caused by a highly resistant strain of Klebsiella pneumoniae occurs in a 13-bed intensive care unit (ICU). Each patient in the ICU is in a private, enclosed room. Which of the following infection control measures is most likely to decrease the spread of this organism to other patients in the ICU and elsewhere in the hospital? A) Place infected patients in rooms with at least six air exchanges per hour B) Place infected patients in rooms with negative air pressure C) Place infected patients in rooms with positive air pressure D) Promote thorough hand cleansing before and after medical personnel visit each patient E) Require medical personnel to wear masks when visiting infected patients

National Board o f Medical Examiners - Google Chrome Q https:/?www.starttest.com/api/5.1.1.0/TTDStart.aspx?SVC=a2401610-d3a2-45ec-83a7-b56dlda82cf6

Exam Section 4: Item 21 o f 50 I Mark

N ational Board o f M edical E xam iners C om prehensive Basic Science Self-Assessment

T im e R em aining: 3 hr 53 m in 55 sec

21. A 39-year-old man comes to the physician because he has had an intense, stabbing pain behind his left eye for 5 nights. The headaches generally begin 1 hour after he goes to bed and last for about 1 hour. During the pain, he has left-sided nasal stuffiness and tearing from his left eye. He had a similar series of nightly headaches for 6 weeks several years ago. Which of the following is the most likely diagnosis? o A) Cluster headache O B) Factitious disorder C) Generalized anxiety disorder O D) Sinusitis O E) Temporal arteritis O F) Tension-type headache

Previous

Lab Values

Review

Help

Pause

O
4:21 PM
* ..ill ><>

I)

&

rSTEaI

rfcjrxy' r^i

28/08/2012

National Board o f Medical Examiners - Google Chrome Q https:/?www.starttest.com/api/5.1.1.0/TTDStart.aspx?SVC=a2401610-d3a2-45ec-83a7-b56dlda82cf6

Exam Section 4: Item 22 o f 50 I Mark

N ational Board o f M edical E xam iners C om prehensive Basic Science Self-Assessment

T im e R em aining: 3 hr 53 m in 13 sec

22. A 48-year-old woman with ductal carcinoma of the right breast has a modified radical mastectomy. After the operation, she has winging of the scapula when she presses her outstretched arms against a wall. Which of the following nerves is most likely injured? O A) Axillary o B) Long thoracic
0

C) Lower subscapular D) Suprascapular E) Thoracodorsal

Previous

Lab Values

Review

&
Help
*

Pause

O
4:22 PM
..ill ><>

I)

&

rSTEaI

rfcjrxy' r^i

28/08/2012

National Board o f Medical Examiners - Google Chrome Q h ttp s://w w w .sta rtte st.e o m /a p i/5 .l.1.0/TTDStart.aspx?SVC=a2401610-d3a2-45ec-83a7-b56dlda82cf6

1 cz, I IB 1 ^ 3 - 1

Exam Section 4: Item 23 o f 50 M ark

N ational Board o f M edical E xam iners C om prehensive Basic Science Self-Assessment

T im e R em aining: 3 hr 53 m in 10 sec

23. A 50-year-old man with a long-standing history of type 2 diabetes mellitus comes to the physician because of several fainting episodes and fatigue during the past month. He appears chronically ill. His temperature is 37C (98.6F), pulse is 95/min, respirations are 17/min, and blood pressure is 155/95 mm Hg. Physical examination shows pallor. Laboratory studies show: Hemoglobin Hematocrit Mean corpuscular hemoglobin Mean corpuscular hemoglobin concentration Mean corpuscular volume Leukocyte count Platelet count Serum Urea nitrogen Creatinine Total bilirubin Which of the following is the most appropriate pharmacotherapy for this patient? O A) Cyclosporine O B) Deferoxamine O C) Erythropoietin D)
0 0

8 g/dL 24% 30 pg/cell 33% Hb/cell 85 pm3 8000/mm3 200,000/mm3 60 mg/dL 6 mg/dL 0.9 mg/dL

Filgrastim

E) Folic acid F)Oprelvekin G) Vitamin B 12(cyanocobalamin)

r 0

National Board o f Medical Examiners - Google Chrome https:/.'www.starttest.com/api/5.1.1.0/ITDStart.aspx?SVC=a2401610-d3a2-45ec-83a7-b56dlda82cf6

Exam Section 4: Item 24 o f 50 I Mark

N ational Board o f M edical E xam iners C om prehensive Basic Science Self-Assessment

T im e R em aining: 3 hr 51 m in 55 sec

24.

An investigator studying the molecular events in the underlying causes of an autoimmune disease develops a collection of monoclonal antibodies to ribosomal proteins. One of these monoclonal antibodies is found to inhibit the ribosome peptidyl transferase. Which of the following is most likely affected in the presence of this antibody? A) Assembly of 80S ribosomal subunits B) Attachment of amino acids to tRNA C) Dissociation of 80S ribosomes into 60S and 40S subunits D) Establishment of covalent bonding between amino acids

O E) Release of the mRNA from ribosomes

r 0

National Board o f Medical Examiners - Google Chrome https:/.'www.starttest.com/api/5.1.1.0/ITDStart.aspx?SVC=a2401610-d3a2-45ec-83a7-b56dlda82cf6

Exam Section 4: Item 25 o f 50 I Mark

N ational Board o f M edical E xam iners C om prehensive Basic Science Self-Assessment

T im e R em aining: 3 hr 46 m in 34 sec

25.

A 15-year-old boy is brought to the physician by his father because of behavioral changes during the past month. He has been failing his classes at school and staying out late at night. His father reports that the family moved to a new neighborhood 2 months ago, and that the local teenagers are "all drug addicts and criminals." His pulse is 110/min, respirations are 24/min, and blood pressure is 140/85 mm Hg. Physical examination shows pupillary dilation and diaphoresis. On mental status examination, his affect is agitated, and his thought process is confused. Intoxication with which of the following substances is the most likely cause of this patient's symptoms?

O A) Alcohol B)
0

Amphetamine Cannabis

C) Amyl nitrate D) E) Heroin

National Board o f Medical Examiners - Google Chrome Q https:/?www.starttest.com/api/5.1.1.0/TTDStart.aspx?SVC=a2401610-d3a2-45ec-83a7-b56dlda82cf6

Exam Section 4: Item 26 o f 50 I Mark

N ational Board o f M edical E xam iners C om prehensive Basic Science Self-Assessment

T im e R em aining: 3 hr 45 m in 8 sec

26. Homologous pairing of chromosomes in prophase I is essential for which of the following normal processes? O A) Centromere division O B) Chromosome uncoiling O C) Crossing over O D) DNA replication

Previous

Lab Values

Review

Help

Pause

O
4:30 PM
* ..ill ><>

I)

&

rSTEaI

rfcjrxy' r^i

28/08/2012

r 0

National Board o f Medical Examiners - Google Chrome https:/.'www.starttest.com/api/5.1.1.0/ITDStart.aspx?SVC=a2401610-d3a2-45ec-83a7-b56dlda82cf6

Exam Section 4: Item 27 o f 50 I Mark

N ational Board o f M edical E xam iners C om prehensive Basic Science Self-Assessment

T im e R em aining: 3 hr 45 m in 3 sec

27. A 9-year-old boy is brought to the emergency department because of fever, chest pain, and migratory joint pain of his shoulders, hips, and knees for 2 days. Four weeks ago, he had a febrile pharyngitis, which resolved 2 weeks later without treatment. His temperature is 39C (102.2F). The lungs are clear to auscultation. A pericardial friction rub and quiet heart sounds are heard. Throat cultures do not grow any pathogens. Laboratory testing shows increased antibody titers to streptolysin 0 . The greatest risk for death at this time is from which of the following? O A) Aortic stenosis O B) Embolism C) Mitral insufficiency O D) Myocarditis E) Septic shock

r 0

National Board o f Medical Examiners - Google Chrome https:/.'www.starttest.com/api/5.1.1.0/ITDStart.aspx?SVC=a2401610-d3a2-45ec-83a7-b56dlda82cf6

Exam Section 4: Item 28 o f 50 I Mark

N ational Board o f M edical E xam iners C om prehensive Basic Science Self-Assessment

T im e R em aining: 3 hr 44 m in 13 sec

28. A previously healthy 40-year-old man is brought to the emergency department 1 hour after the sudden onset of severe pain in his left leg while playing tennis. He is found to have ruptured the left Achilles tendon and undergoes operative repair and long leg cast immobilization. Six months later, the left calf shows a 2-cm decrease in circumference compared with the right calf. Which of the following is the most likely cause of this decrease? A) Decreased glycogen synthesis B) Decreased myosin light chain phosphatase activity C) Increased phosphatidyl degradation D) Increased protein degradation E) Mitochondria damage O F) Necrosis of muscle fibers

r 0

National Board o f Medical Examiners - Google Chrome https:/..'www.starttest.com/api/5.1.1.0/ITDStart.aspx?SVC=a2401610-d3a2-45ec-83a7-b56dlda82cf6

Exam Section 4: Item 29 o f 50 I Mark

N ational Board o f M edical E xam iners C om prehensive Basic Science Self-Assessment

T im e R em aining: 3 hr 42 m in 49 sec

29. A 52-year-old man comes to the physician because of a 3-month history of epigastric abdominal pain; he also has had an unintentional 6.8-kg (15-lb) weight loss during this period. He has osteoarthritis treated with naproxen as needed. He immigrated to the USA from Japan 6 months ago. He eats mostly traditional Japanese food prepared by his wife. He has smoked 2 packs of cigarettes daily for 30 years and drinks three to four glasses of wine daily. He is 170 cm (5 ft 7 in) tall and now weighs 82 kg (180 lb); BMI is 28 kg/m2. Physical examination shows epigastric tenderness. Upper gastrointestinal endoscopy shows a 4-cm ulcer in the stomach. Examination of a biopsy specimen of the lesion confirms adenocarcinoma. Which of the following is the strongest predisposing risk factor for this patient's condition? A) Alcohol use o B) Diet O C) Ethnicity D) Naproxen use E) Tobacco use

National Board o f Medical Examiners - Google Chrome Q https:/?www.starttest.com/api/5.1.1.0/TTDStart.aspx?SVC=a2401610-d3a2-45ec-83a7-b56dlda82cf6

Exam Section 4: Item 30 o f 50 I Mark

N ational Board o f M edical E xam iners C om prehensive Basic Science Self-Assessment

T im e R em aining: 3 hr 42 m in 43 sec

30.

A 57-year-old man dies of a myocardial infarction. At autopsy, examination of lung tissue shows alveoli containing epithelial cells that are cuboidal in shape with electron dense lamellar bodies 1 m in diameter. These cells most likely produce which of the following? 2

O A) Collagen, type I B) Epinephrine O C) Histamine O D)lgG O E) Surfactant

Previous

Lab Values

Review

&
Help

Pause

O
4:32 PM 28/08/2012

I)

&

rSTEaI

rfcjrxy' r^i

r 0

National Board o f Medical Examiners - Google Chrome https:/.'www.starttest.com/api/5.1.1.0/ITDStart.aspx?SVC=a2401610-d3a2-45ec-83a7-b56dlda82cf6

Exam Section 4: Item 31 o f 50 I Mark

N ational Board o f M edical E xam iners C om prehensive Basic Science Self-Assessment

T im e R em aining: 3 hr 42 m in 21 sec

31. A 63-year-old man comes to the emergency department 30 minutes after the sudden onset of right-sided chest pain and shortness of breath that began after a 10-hour drive in his truck. The pain is more severe with inspiration. He does not smoke cigarettes. His pulse is 110/min, and respirations are 32/min. The lungs are clear to auscultation. Which of the following findings is most likely on ventilation-perfusion lung scans? A) Absence of perfusion in some areas that are ventilated B) Absence of ventilation in some areas that are perfused C) Absence of ventilation-perfusion mismatching D) Decreased perfusion gradient from the apices to the bases of the lungs E) Increased ventilation-perfusion ratios at lung bases compared with those at the apices

r 0

National Board o f Medical Examiners - Google Chrome https:/..'www.starttest.com/api/5.1.1.0/ITDStart.aspx?SVC=a2401610-d3a2-45ec-83a7-b56dlda82cf6

Exam Section 4: Item 32 o f 50 I Mark

N ational Board o f M edical E xam iners C om prehensive Basic Science Self-Assessment

T im e R em aining: 3 hr 40 m in 38 sec

32. A 55-year-old man is brought to the emergency department 45 minutes after he collapsed at home. His pulse is 120/min, respirations are 16/min, and blood pressure is 80/45 mm Hg. Physical examination shows no other abnormalities. His hemoglobin concentration is 8 g/dL, hematocrit is 24%, and platelet count is 240,000/mm3 Endoscopy shows a bleeding . duodenal ulcer, which is surgically repaired. He receives 4 units of packed red blood cells. Two days later, laboratory studies show: Hemoglobin Hematocrit Serum Urea nitrogen Creatinine Urine Blood Sediment Volume Total protein Sodium Fractional excretion of sodium Osmolality 10 g/dL 30% 70 mg/dL 3 mg/dL absent granular casts 100 ml_/24 h 100 mg/24 h 50 mEq/L 2.5% 300 mOsmol/kg H20

Two weeks later, the urine sediment is clear and renal function is normal. Histologic examination of a renal biopsy specimen is most likely to show which of the following at this time? A) Coagulation necrosis B) Collagen scar C) Granulation tissue D) Lymphocytic infiltrate and fibrogenesis E) Regenerating tubular epithelial cells

r 0

National Board o f Medical Examiners - Google Chrome https:/.'www.starttest.com/api/5.1.1.0/ITDStart.aspx?SVC=a2401610-d3a2-45ec-83a7-b56dlda82cf6

Exam Section 4: Item 33 o f 50 I Mark

N ational Board o f M edical E xam iners C om prehensive Basic Science Self-Assessment

T im e R em aining: 3 hr 38 m in 42 sec

33. Twenty subjects participate in a study of cardiopulmonary physiology. The pulmonary circulation of these subjects most likely has which of the following characteristics compared with their bronchial circulation? A) Greater number of branches supplying the visceral pleura B) Larger number of anastomoses with the intercostal arteries C) Larger percentage of cardiac output D) Larger volume of nutrient blood to the conducting airways O E) Lower-volume, higher-pressure system

r 0

National Board o f Medical Examiners - Google Chrome https:/.'www.starttest.com/api/5.1.1.0/ITDStart.aspx?SVC=a2401610-d3a2-45ec-83a7-b56dlda82cf6

Exam Section 4: Item 34 o f 50 I Mark

N ational Board o f M edical E xam iners C om prehensive Basic Science Self-Assessment

T im e R em aining: 3 hr 36 m in 46 sec

34. A 40-year-old man is brought to the emergency department after being involved in a motor vehicle collision. He is pronounced dead on arrival. He had a history of fever, headache, chills, and pain in the right upper quadrant of the abdomen 5 days ago. During autopsy, histologic examination of a liver section shows disruption of the normal hepatic lobule. Small shrunken hepatocytes with intense eosinophilic cytoplasm, fragmented nuclear chromatin, and cytoplasmic bleb formations are noted. Which of the following processes is most likely occurring in the hepatocytes described? O A)Apoptosis O B) Atrophy O C) Caseous necrosis D) Coagulation necrosis E) Dysplasia F) Fatty change G) Heterophagy H) Liquefaction necrosis I) Metaplasia

National Board o f Medical Examiners - Google Chrome Q h ttp s://w w w .sta rtte st.e o m /a p i/5 .l.1.0/TTDStart.aspx?SVC=a2401610-d3a2-45ec-83a7-b56dlda82cf6

1 cz, I IB 1 ^ 3 - 1

Exam Section 4: Item 35 o f 50 M ark

N ational Board o f M edical E xam iners C om prehensive Basic Science Self-Assessment

T im e R em aining: 3 hr 36 m in 1 sec

35. The tracing shows contractile responses of a skeletal muscle preparation to electrical stimulation at five different frequencies. The amount of calcium sequestration in sarcoplasmic reticulum is highest during which of the following labeled responses?

A)

B)

C)

D)

E)

r 0

National Board o f Medical Examiners - Google Chrome https:/.'www.starttest.com/api/5.1.1.0/ITDStart.aspx?SVC=a2401610-d3a2-45ec-83a7-b56dlda82cf6

Exam Section 4: Item 36 o f 50 I Mark

N ational Board o f M edical E xam iners C om prehensive Basic Science Self-Assessment

T im e R em aining: 3 hr 35 m in 25 sec

36. A 26-year-old primigravid woman delivers a 2948-g (6-lb 8-oz) male newborn at term. Six months ago, she received a vaccine, Streptococcus agalactiae (group B), serotype III, during a clinical trial. Screening indicated that she had generated an antibody response to this polysaccharide antigen within 1 month of the vaccination. This antibody was detected in her serum immediately prior to the onset of labor. Eight hours after delivery, the newborn appears progressively lethargic, and his temperature is 38.2C (100.8F). Cerebrospinal fluid and blood cultures grow S. agalactiae (group B), serotype III. It is suspected that the mother produced only one antibody isotype in response to the vaccine. This isotype is most likely which of the following? A) IgA C B)lgD C) IgE C D)lgG E)lgM

Previous

Lab Values

Review

Pause 4:40 PM 28/08/2012

r 0

National Board o f Medical Examiners - Google Chrome https:/.'www.starttest.com/api/5.1.1.0/ITDStart.aspx?SVC=a2401610-d3a2-45ec-83a7-b56dlda82cf6

Exam Section 4: Item 37 o f 50 I Mark

N ational Board o f M edical E xam iners C om prehensive Basic Science Self-Assessment

T im e R em aining: 3 hr 34 m in 46 sec

37.

A 4-year-old girl has had "puffy" eyes for the past 3 days. She has had an upper respiratory tract infection for the past week. A urine dipstick test shows 3+ protein. Urinalysis shows no red or white blood cells or casts. Which of the following mechanisms is the most likely cause of this patient's condition? A) Attack by antiglomerular basement membrane antibodies

O B) Damage to the loop of Henle C) Damage to the proximal renal tubule D) Loss of negative glomerular charge O E) Presence of a posterior urethral valve

r 0

National Board o f Medical Examiners - Google Chrome https:/..'www.starttest.com/api/5.1.1.0/ITDStart.aspx?SVC=a2401610-d3a2-45ec-83a7-b56dlda82cf6

Exam Section 4: Item 38 o f 50 I Mark

N ational Board o f M edical E xam iners C om prehensive Basic Science Self-Assessment

T im e R em aining: 3 hr 34 m in 15 sec

38. A 25-year-old man with cystic fibrosis comes to the physician because of a 3-day history of fever, cough, and progressive shortness of breath. His temperature is 38C (100.4F), and respirations are 32/min. Bilateral crackles and rhonchi are heard on auscultation of the chest. A nutrient agar plate seeded with bronchial secretions grows an aerobic, oxidase-positive, gram-negative rod that forms slimy colonies and produces pyocyanin. The organism does not ferment lactose. Treatment with multiple appropriate antibiotics has not led to eradication of the causal organism. Which of the following is the most likely explanation for the persistent colonization of this patient by this organism? A) Bacteriophage superinfection B) Biofilm formation in the lower respiratory tract C) Overgrowth of acapsular strains O D) Pyocyanin synthesis O E) Synthesis of exotoxin A

r 0

National Board o f Medical Examiners - Google Chrome https:/.'www.starttest.com/api/5.1.1.0/ITDStart.aspx?SVC=a2401610-d3a2-45ec-83a7-b56dlda82cf6

Exam Section 4: Item 39 o f 50 I Mark

N ational Board o f M edical E xam iners C om prehensive Basic Science Self-Assessment

T im e R em aining: 3 hr 32 m in 10 sec

39.

A 10-year-old boy who has had type 1 diabetes mellitus for 1 year is receiving insulin. One hour after his morning dose of insulin, he becomes tremulous and diaphoretic and has tachycardia. Several hours later, his symptoms resolve. His blood glucose concentration is now increased. Which of the following is the most likely cause of this patient's hyperglycemia? A) Activation of hepatic adenylyl cyclase B) Activation of muscle glycogen synthase C) Activation of muscle phosphorylase D) Activation of muscle protein phosphatase

O E) Inhibition of hepatic protein kinase A

r 0

National Board o f Medical Examiners - Google Chrome https:/.'www.starttest.com/api/5.1.1.0/ITDStart.aspx?SVC=a2401610-d3a2-45ec-83a7-b56dlda82cf6

Exam Section 4: Item 40 o f 50 I Mark

N ational Board o f M edical E xam iners C om prehensive Basic Science Self-Assessment

T im e R em aining: 3 hr 29 m in 46 sec

40. A 16-year-old boy has a 6-month history of type 1 diabetes mellitus and requires daily insulin injections. His blood glucose control has been poor recently, which the physician suspects may be related to the patient's reluctance to self-administer insulin. Which of the following questions is most appropriate to broach this issue with the patient? A) "Giving yourself insulin injections can be difficult. What's it like for you?" B) "I get the feeling you have not been taking your insulin regularly. Would you like me to arrange some training for you by our nurses?" C) "It's been my experience that most poorly controlled diabetics have trouble giving themselves insulin. Do you have this problem?" D) "You are clearly having difficulty with insulin injections. Would you like to arrange for a family member to give you your shots?" O E) "You seem to have missed your shots. How often has this happened?"

r 0

National Board o f Medical Examiners - Google Chrome https:/..'www.starttest.com/api/5.1.1.0/ITDStart.aspx?SVC=a2401610-d3a2-45ec-83a7-b56dlda82cf6

Exam Section 4: Item 41 o f 50 I Mark

N ational Board o f M edical E xam iners C om prehensive Basic Science Self-Assessment

T im e R em aining: 3 hr 28 m in 51 sec

41.

A 50-year-old man comes to the physician because of a 2-month history of pain of his wrists, changes in skin color, and progressive fatigue. His brother has type 2 diabetes mellitus and cirrhosis. Physical examination shows bronze-colored skin, tenderness of the metacarpophalangeal joints in both hands, and hepatosplenomegaly. Serum studies show: AST ALT Ferritin Total iron-binding capacity Transferrin saturation 100 U/L 110 U/L 1200ng/mL 200 pg/dL (N=250-400) 80% (N=20-50)

Analysis of a liver biopsy specimen shows a markedly increased iron concentration and cirrhosis. Which of the following is the most likely cause of the findings in this patient? O A) Increased erythropoietin action B) Increased intestinal iron absorption C) Increased oral iron intake O D) Decreased erythropoiesis O E) Decreased iron excretion F) Decreased serum transferrin concentration

National Board o f Medical Examiners - Google Chrome Q h ttp s://w w w .sta rtte st.e o m /a p i/5 .l.1.0/TTDStart.aspx?SVC=a2401610-d3a2-45ec-83a7-b56dlda82cf6

Exam Section 4: Item 42 o f 50 H Mark

National Board o f Medical Examiners Comprehensive Basic Science Self-Assessment

Tim e Remaining: 3 hr 28 min 11 sec

42. An autopsy is done on a 46-year-old woman who died of adenocarcinoma of the colon. Examination of the neck shows a 5-cm, rounded mass next to the bifurcation of the carotid artery. A section of the mass is shown in the photomicrograph. Immunohistochemistry of the section is positive for synaptophysin, chromogranin and neuron-specific enolase. Electron microscopy shows numerous electron-dense, membrane-bound neurosecretory granules. Examination of the adrenal glands shows no masses. Which of the following is the most likely diagnosis? A) Metastatic colonic adenocarcinoma B) Metastatic squamous cell carcinoma of the larynx C) Papillary carcinoma of the thyroid gland D) Paraganglioma E) Parathyroid adenoma

'V

ill

ft

7.
1

r 0

National Board o f Medical Examiners - Google Chrome https:/.'www.starttest.com/api/5.1.1.0/ITDStart.aspx?SVC=a2401610-d3a2-45ec-83a7-b56dlda82cf6

Exam Section 4: Item 43 o f 50 I Mark

N ational Board o f M edical E xam iners C om prehensive Basic Science Self-Assessment

T im e R em aining: 3 hr 26 m in 23 sec

43. A 29-month-old boy is brought to the physician by his mother because of a 2-week history of a green nasal discharge and cough. He has a past history of frequent respiratory tract illnesses and pneumonia. The mother states that her son has a 6-month history of large, foul-smelling stools and a salty taste when she kisses him. There is no family history of a similar condition. He is at the 5th percentile for height and below the 3rd percentile for weight. His temperature is 38.3C (100.9F). Abdominal examination shows distention. Genetic testing shows a mutation in the Phe residue at position 508 in the cystic fibrosis transmembrane conductance regulator gene. Which of the following types of modification is required prior to destruction of misfolded proteins in this patient's proteasomes? O A) Dimerization B) Glycosylation O C) Phosphorylation O D) Sulfation O E) Ubiquitination

r 0

National Board o f Medical Examiners - Google Chrome https:/..'www.starttest.com/api/5.1.1.0/ITDStart.aspx?SVC=a2401610-d3a2-45ec-83a7-b56dlda82cf6

Exam Section 4: Item 44 o f 50 0 M ark

N ational Board o f M edical E xam iners C om prehensive Basic Science Self-Assessment

T im e R em aining: 3 hr 25 m in 17 sec

44. A 60-year-old man comes to the physician because of a 2-week history of fatigue and exercise intolerance. One month ago, he was diagnosed with stage I hypertension and began treatment with propranolol. His pulse is 56/min, compared with 72/min 1 month ago. His treatment is switched to pindolol. His symptoms resolve within 1 month, and his pulse is 68/min. Which of the following mechanisms of action of pindolol best explains its effect on this patient's heart rate? O A) Competitive agonism B) Competitive antagonism O C) Inverse agonism o D) Partial agonism

r 0

National Board o f Medical Examiners - Google Chrome https:/.'www.starttest.com/api/5.1.1.0/ITDStart.aspx?SVC=a2401610-d3a2-45ec-83a7-b56dlda82cf6

Exam Section 4: Item 45 o f 50 I Mark

N ational Board o f M edical E xam iners C om prehensive Basic Science Self-Assessment

T im e R em aining: 3 hr 24 m in 59 sec

45. A 24-year-old man comes to the physician with his 25-year-old wife because they have been unable to conceive for 3 years. The wife has been evaluated for infertility, and the test results were normal. Analysis of the mans semen shows normal numbers of living sperm but they are immotile. In addition to infertility, this man is most likely to have which of the following associated conditions? A) Cholelithiasis B) Coronary artery disease C) Fat malabsorption D) Glomerulonephritis O E) Sinusitis

r 0

National Board o f Medical Examiners - Google Chrome https:/..'www.starttest.com/api/5.1.1.0/ITDStart.aspx?SVC=a2401610-d3a2-45ec-83a7-b56dlda82cf6

Exam Section 4: Item 46 o f 50 I Mark

N ational Board o f M edical E xam iners C om prehensive Basic Science Self-Assessment

T im e R em aining: 3 hr 23 m in 51 sec

46. A 72-year-old woman comes to the emergency department because of bloody diarrhea for the past 12 hours and diffuse abdominal pain for the past 4 days. History includes a cerebral infarction 3 years ago from which she recovered completely. Examination shows watery stool with red blood. Colonoscopy shows patchy ulceration at the area near the splenic flexure. Which of the following is the most likely diagnosis? A) Celiac sprue B) D) Irritable bowel syndrome Pancreatic insufficiency O C) Ischemic colitis E) Regional enteritis F) Ulcerative colitis

r 0

National Board o f Medical Examiners - Google Chrome https:/.'www.starttest.com/api/5.1.1.0/ITDStart.aspx?SVC=a2401610-d3a2-45ec-83a7-b56dlda82cf6

Exam Section 4: Item 47 o f 50 I Mark

N ational Board o f M edical E xam iners C om prehensive Basic Science Self-Assessment

T im e R em aining: 3 hr 23 m in 15 sec

47. A male newborn delivered at 34 weeks gestation by an uncomplicated spontaneous vaginal delivery develops respiratory distress 30 minutes later. His respirations are 40/min. Physical examination shows cyanosis and lower rib retractions with respiration. Chest x-rays show fine densities with a reticulogranular appearance bilaterally. Which of the following is the most likely cause of these findings? A) Erythroblastosis fetalis B) Extralobar sequestration C) Intracranial hemorrhage D) Pulmonary surfactant deficiency O E) Situs inversus

r 0

National Board o f Medical Examiners - Google Chrome https:/.'www.starttest.com/api/5.1.1.0/ITDStart.aspx?SVC=a2401610-d3a2-45ec-83a7-b56dlda82cf6

Exam Section 4: Item 48 o f 50 I Mark

N ational Board o f M edical E xam iners C om prehensive Basic Science Self-Assessment

T im e R em aining: 3 hr 22 m in 42 sec

48. A 23-year-old woman is brought to the emergency department 45 minutes after sustaining an injury to her right shoulder when she was forcefully thrown into a swimming pool. The patient is holding her right arm in external rotation. There is an area of tenderness and absent sensation over the lateral aspect of the forearm. Immediately after reduction of the glenohumeral joint, she is unable to flex the elbow against gravity. Which of the following structures is most likely injured in this patient? O A) Axillary nerve B) Musculocutaneous nerve C) Posterior cord of the brachial plexus O D) Radial nerve O E) Thoracodorsal nerve

r 0

National Board o f Medical Examiners - Google Chrome https:/..'www.starttest.com/api/5.1.1.0/ITDStart.aspx?SVC=a2401610-d3a2-45ec-83a7-b56dlda82cf6

Exam Section 4: Item 49 o f 50 I Mark

N ational Board o f M edical E xam iners C om prehensive Basic Science Self-Assessment

T im e R em aining: 3 hr 21 m in 25 sec

49. A 33-year-old woman who is right-handed is brought to the physician because of a 3-day history of progressive weakness and numbness of her arms and legs. Neurologic examination shows proximal and distal weakness of the upper and lower extremities. There is areflexia. Sensation to vibration and joint position is decreased in the fingers and toes. Nerve conduction studies show a slow conduction velocity in the median, ulnar, peroneal, and tibial nerves. These electrophysiologic findings most likely indicate impaired function of which of the following ion channels? A) Neurotransmitter-gated Ca2+ channels B) Neurotransmitter-gated K +channels C) Neurotransmitter-gated Na+channels D) Voltage-gated C a2+ channels E) Voltage-gated K +channels F) Voltage-gated Na+channels

r 0

National Board o f Medical Examiners - Google Chrome https:/.'www.starttest.com/api/5.1.1.0/ITDStart.aspx?SVC=a2401610-d3a2-45ec-83a7-b56dlda82cf6

Exam Section 4: Item 50 o f 50 I Mark

N ational Board o f M edical E xam iners C om prehensive Basic Science Self-Assessment

T im e R em aining: 3 hr 18 m in 27 sec

50. A study is conducted to evaluate the onset and course of cardiovascular disease in African American men and women in the USA. Census data from the year 2000 are used to select the study sample. A series of factors including family income and years of education is defined. Census tracts most representative of the national census data for the factors selected are used for sampling. Which of the following best describes this sampling technique? A) Blocked randomization O B) Population-based O C) Sample of convenience O D) Self-selection O E) Sequential

Das könnte Ihnen auch gefallen